nclex GU, Pediatric GU questions Nclex, renal gu nclex, Renal & GU- NCLEX, GU NCLEX 3500, NCLEX GU

अब Quizwiz के साथ अपने होमवर्क और परीक्षाओं को एस करें!

The nurse is reinforcing teaching with the caregiver of 5-year-old twins regarding urinary tract infections (UTI's). The caregiver is puzzled about why her daughter has had three urinary tract infections but her son has had none. She reports that their diets and fluid intake is similar. Which of the following statements would be accurate for the nurse to tell this mother? a) "It is unlikely that your daughter is practicing good cleaning habits after she voids." b) "Girls need more vitamin C than boys to keep their urinary tract healthy, so your daughter may be deficient in vitamin C." c) "A girl's urethra is much shorter and straighter than a boy's, so it can be contaminated fairly easily." d) "The position of the urethra in girls makes girls more susceptible than boys to UTI's."

"A girl's urethra is much shorter and straighter than a boy's, so it can be contaminated fairly easily." Correct Explanation: Many different bacteria may infect the urinary tract, and intestinal bacteria, particularly Escherichia coli, account for about 80% of acute episodes. The female urethra is shorter and straighter than the male urethra, so it is more easily contaminated with feces.

infuse a bolus of normal saline.

A patient complains of leg cramps during hemodialysis. The nurse should first _____________

Place a bedside commode near the patient's bed.

A patient in the hospital has a history of functional urinary incontinence. Which nursing action will be included in the plan of care?

1000

A patient in the oliguric phase of acute renal failure has a 24-hour fluid output of 150 mL emesis and 250 mL urine. The nurse plans a fluid replacement for the following day of ___ mL.

Recent weight gain

A patient is admitted to the hospital with new onset nephrotic syndrome. Which assessment data will the nurse expect to find related to this illness?

is much less likely to clot.

A patient needing vascular access for hemodialysis asks the nurse what the differences are between an arteriovenous (AV) fistula and a graft. The nurse explains that one advantage of the fistula is that it

Obtain a midstream urine specimen for culture and sensitivity testing.

A patient returns to the clinic with recurrent dysuria after being treated with trimethoprim and sulfamethoxazole (Bactrim) for 3 days. Which action will the nurse plan to take?

Urine output is 20 mL/hr for 2 hours.

A patient undergoes a nephrectomy after having massive trauma to the kidney. Which assessment finding obtained postoperatively is most important to communicate to the surgeon?

Cardiac rhythm

A patient who has acute glomerulonephritis is hospitalized with acute kidney injury (AKI) and hyperkalemia. Which information will the nurse obtain to evaluate the effectiveness of the prescribed calcium gluconate IV?

Catheterization technique and schedule

A patient who has bladder cancer had a cystectomy with creation of an Indiana pouch. Which topic will be included in patient teaching?

The patient has noticed clots in the urine.

A patient who has had a transurethral resection with fulguration for bladder cancer 3 days previously calls the nurse at the urology clinic. Which information given by the patient is most important to report to the health care provider?

Excess fluid volume related to low serum protein levels

A patient who is diagnosed with nephrotic syndrome has 3+ ankle and leg edema and ascites. Which nursing diagnosis is a priority for the patient?

The patient describes allergies to shellfish and penicillin.

A patient who is scheduled for an intravenous pyelogram (IVP) gives the nurse the following information. Which information has the most immediate implications for the patient's care?

Insert retention catheter.

A patient with a history of benign prostatic hyperplasia (BPH) is admitted with acute urinary retention and an elevated blood urea nitrogen (BUN) and creatinine. Which of these prescribed therapies should the nurse implement first?

Ask the patient about use of any medications.

A patient with a possible urinary tract infection (UTI) gives the nurse in the clinic a urine specimen that is a red-orange color. Which action should the nurse take first?

rapid respirations.

A patient with acute kidney injury (AKI) has an arterial blood pH of 7.30. The nurse will assess the patient for _________________

a Fleet enema.

A patient with an elevated blood urea nitrogen (BUN) and serum creatinine is scheduled for a renal arteriogram. The nurse should question an order from radiology for bowel preparation with the use of

the need to empty the bladder before treatment.

A patient with bladder cancer is scheduled for intravesical chemotherapy. In preparation for the treatment the nurse will teach the patient about ______________

Milk of magnesia 30 mL

A patient with chronic kidney disease (CKD) brings all home medications to the clinic to be reviewed by the nurse. Which medication being used by the patient indicates that patient teaching is required?

Apply a pressure dressing and keep the patient on the affected side for 30 to 60 minutes.

A patient with diabetic nephropathy is admitted for a right renal biopsy. Immediately after the biopsy, which of these is an essential nursing action?

potassium.

A patient with hypertension and stage 2 chronic kidney disease (CKD) is receiving captopril (Capoten). Before administration of the medication, the nurse will check the patient's ___________________

anticoagulants.

A patient with nephrotic syndrome develops flank pain. The nurse will anticipate teaching the patient about treatment with ______________

Administer prescribed analgesics.

A patient with renal calculi is hospitalized with gross hematuria and severe colicky left flank pain. Which nursing action will be of highest priority at this time?

maintaining cardiac output.

A patient with severe heart failure develops elevated blood urea nitrogen (BUN) and creatinine levels. The nurse will plan care to meet the goal of __________________

Ibuprofen (Advil) 400 mg PO PRN for pain

A patient with stage 2 chronic kidney disease (CKD) is scheduled for an intravenous pyelogram (IVP). Which of these orders for the patient will the nurse question?

organ meats and sardines.

A patient's renal calculus is analyzed as being very high in uric acid. To prevent recurrence of stones, the nurse teaches the patient to avoid eating __________________

check which medications the patient is currently taking.

A patient's urine dipstick indicates a small amount of protein in the urine. The next action by the nurse should be to ______________

A patient with a history of recurrent urinary tract infections has been scheduled for a cystoscopy. Which teaching point should you emphasize before the procedure? A. "You may have pink-tinged urine and a burning sensation after the cystoscopy." B. "You'll need to refrain from eating or drinking after midnight the day before the test." C. "You'll require a urinary catheter inserted before the cystoscopy, and it will be in place for a few days." D. "The morning of the test, the nurse will ask you to drink some water that contains a contrast solution."

A. "You may have pink-tinged urine and a burning sensation after the cystoscopy." Pink-tinged urine, a burning sensation, and frequency are common after a cystoscopy. The patient does not need to be NPO before the test, and a contrast solution is unnecessary. Cystoscopy does not always necessitate catheterization before or after the procedure.

Which nursing diagnosis is a priority in the care of a patient with renal calculi? A. Acute pain B. Deficient fluid volume C. Risk for constipation D. Risk for powerlessness

A. Acute pain Urinary stones are associated with severe abdominal or flank pain. Deficient fluid volume is unlikely to result from urinary stones, and constipation is more likely to be an indirect consequence rather than a primary clinical manifestation of the problem. The presence of pain supersedes powerlessness as an immediate focus of nursing care.

A patient is admitted to the hospital with severe renal colic caused by renal lithiasis. What is your first priority in management of the patient? A. Administer opioids as prescribed. B. Obtain supplies for straining all urine. C. Encourage fluid intake of 3 to 4 L/day. D. Keep the patient NPO in preparation for surgery.

A. Administer opioids as prescribed. Pain management and patient comfort are primary nursing responsibilities when managing an obstructing stone and renal colic.

What instructions would you give to a patient being treated with nitrofurantoin (Furadantin, Macrodantin)? A. Avoid sunlight and wear sunscreen or use protective clothing. B. Avoid concurrent consumption of calcium-containing products. C. Take the medication concurrently with vitamin C. D. Do not be alarmed if urine is red.

A. Avoid sunlight and wear sunscreen or use protective clothing. Nitrofurantoin carries a significant drug alert. The patient should be taught to avoid sunlight, use sunscreen, and wear protective clothing. The patent should notify the health care provider if fever, chills, cough, chest pain, dyspnea, rash, or numbness or tingling of fingers or toes develop.

What is the most common cause of pyelonephritis? A. Escherichia coli B. Staphylococci C. Streptococci D. Klebsiella

A. Escherichia coli Pyelonephritis usually begins with colonization and infection of the lower urinary tract by the ascending urethral route. Bacteria normally found in the intestinal tract, such as E. coli, Proteus, Klebsiella, and Enterobacter species, frequently cause pyelonephritis; the most common cause is E. coli.

Assist the patient to take a 15-minute sitz bath.

Following an open loop resection and fulguration of the bladder, a patient is unable to void. Which nursing action should be implemented first?

Use an ultrasound scanner to check the postvoiding residual.

Following rectal surgery, a patient voids about 50 mL of urine every 30 to 60 minutes. Which nursing action is most appropriate?

The child with nephrotic syndrome who has ascites and difficulty breathing is probably most comfortable sleeping in which position? a) Supine b) Sims' position c) Prone d) Fowler's

Fowler's Correct Explanation: A Fowler's position (sitting upright) allows ascites fluid to settle downward and not press against the diaphragm, compromising breathing.

"I will empty my bladder every 3 to 4 hours during the day."

The nurse determines that instruction regarding prevention of future urinary tract infections (UTIs) for a patient with cystitis has been effective when the patient states, ___________________

Scrambled eggs, English muffin, and apple juice

The nurse has instructed a patient who is receiving hemodialysis about appropriate dietary choices. Which menu choice by the patient indicates that the teaching has been successful?

may experience blood-tinged urine and urinary frequency.

The nurse informs the patient undergoing cystoscopy that following the procedure, the patient _________________

The patient's peritoneal effluent appears cloudy.

The nurse is assessing a patient who is receiving peritoneal dialysis with 2 L inflows. Which information should be reported immediately to the health care provider?

Joint pain

The nurse is caring for a patient who had kidney transplantation several years ago. Which assessment finding may indicate that the patient is experiencing adverse effects to the prescribed corticosteroid?

Disconnecting the catheter from the drainage tube to obtain a specimen

The nurse observes nursing assistive personnel (NAP) taking the following actions when caring for a patient with a retention catheter. Which action requires that the nurse intervene?

identify renal artery or aortic bruits.

The nurse uses auscultation during assessment of the urinary system to __________________

The patient's central venous pressure (CVP) is decreased.

Two hours after a kidney transplant, the nurse obtains all of the following data when assessing the patient. Which information is most important to communicate to the health care provider?

Drop in urine output

When the nurse is caring for a patient who has had left-sided extracorporeal shock wave lithotripsy, which assessment finding is most important to report to the health care provider?

The patient has metastatic lung cancer.

When the nurse is taking a history for a patient who is a possible candidate for a kidney transplant, which information about the patient indicates that the patient is not an appropriate candidate for transplantation?

The patient cleans the catheter while taking a bath every day.

Which action by a patient who is using peritoneal dialysis (PD) indicates that the nurse should provide more teaching about PD?

Call the health care provider if the ureteral catheter output drops suddenly.

Which action will the nurse include in the plan of care for a patient who has had a ureterolithotomy and has a left ureteral catheter and a urethral catheter in place?

Heart rate 102 beats/minute

Which assessment finding for a patient who has had a cystectomy with an ileal conduit the previous day is most important for the nurse to communicate to the physician?

Blood pressure 88/45 mm Hg

Which assessment finding for a patient who has just been admitted with acute pyelonephritis is most important for the nurse to report to the health care provider?

There is a nontender lump in the axilla.

Which data obtained when assessing a patient who had a kidney transplant 8 years ago and who is receiving the immunosuppressants tacrolimus (Prograf), cyclosporine (Sandimmune), and prednisone (Deltasone) will be of most concern to the nurse?

Peripheral and periorbital edema is resolved.

Which finding by the nurse for a patient admitted with glomerulonephritis indicates that treatment has been effective?

Urine output over an 8-hour period is 2500 mL.

Which information about a patient who was admitted 10 days previously with acute kidney injury (AKI) caused by dehydration will be most important for the nurse to report to the health care provider?

Left-sided flank pain

Which information noted by the nurse when caring for a patient with a bladder infection is most important to report to the health care provider?

Calculated glomerular filtration rate (GFR)

Which information will be most useful to the nurse in evaluating improvement in kidney function for a patient who is hospitalized with acute kidney injury (AKI)?

The urine may turn a reddish-orange color.

Which information will the nurse include when teaching the patient with a urinary tract infection (UTI) about the use of phenazopyridine (Pyridium)?

Check blood pressure before starting dialysis.

Which nursing action for a patient who has arrived for a scheduled hemodialysis session is most appropriate for the RN to delegate to a dialysis technician?

Change the ostomy appliance.

Which nursing action should the nurse who is caring for a patient who has had an ileal conduit for several years delegate to nursing assistive personnel (NAP)?

Avoid unnecessary catheterizations.

Which nursing action will be most helpful in decreasing the risk for hospital-acquired infection (HAI) of the urinary tract in patients admitted to the hospital?

Ask about the usual urinary pattern and any measures used for bladder control.

Which of the following actions will the nurse plan to take first when admitting a patient who has a history of neurogenic bladder as a result of a spinal cord injury?

Hemoglobin level 13 g/dL

Which of the following information obtained by the nurse who is caring for a patient with end-stage renal disease (ESRD) indicates the nurse should consult with the health care provider before giving the prescribed epoetin alfa (Procrit)?

Urine output

Which parameter will be most important for the nurse to consider when titrating the IV fluid infusion rate immediately after a patient has had kidney transplantation?

Phosphate level

Which patient information will the nurse plan to obtain in order to determine the effectiveness of the prescribed calcium carbonate (Caltrate) for a patient with chronic kidney disease (CKD)?

"I will measure my urinary output each day to help calculate the amount I can drink."

Which statement by a patient with stage 5 chronic kidney disease (CKD) indicates that the nurse's teaching about management of CKD has been effective?

Document the information on the assessment form.

While assessing a patient's urinary system, the nurse cannot palpate either kidney. Which action should the nurse take next?

The nurse is preparing a 7-year-old girl for discharge after treatment for nephrotic syndrome. Which instructions would the nurse include in the discharge teaching plan for the parents? a) "Let's meet with the dietitian and plan some meals." b) "Here is some written information from the dietitian." c) "She must severely restrict her sodium intake." d) "She should try to avoid protein."

"Let's meet with the dietitian and plan some meals." Correct Explanation: Consultation with a dietitian would be most helpful for meal planning because so many of children's favorite foods are high in sodium. Restricting sodium may not be necessary if the child is not edematous; in addition, the statement does not teach. Protein-rich snacks should be encouraged. The nurse needs to provide the parents with specific instructions, assistance, and resources in addition to simple written instructions.

**Continent urinary diversion pouch nursing implications:

- *client will need to self cath every 4-6 hours

* Chronic Renal Failure TX pharmacological

- antihypertensives - diuretics - erythropoietin - phosphate binders - vitamin D

Glomerulonephritis Nursing Interventions:

- check proteinuria, specific gravity, and color of urine - weigh client daily - ** if client has hypertension, check BP every 2-4 hours - * teach client to perform dipstick urine test to monitor for protein

* Chronic Renal Failure Diagnostics:

- elevated blood sugar & triglycerides - increased serum potassium level - decreased hemoglobin and hematocrit

*Chronic Renal Failure Diet TX

- lose weight - restrict protein - fluid restriction 600-1000 ml - sodium and potassium restriction

kidney transplantation candidates

- younger than 70 & have a life expectancy of at least 2 more years

* colony count of urine that indicates infection

100,000 colonies/ml of urine

bladder cancer.

A 26-year-old patient who is employed as a hairdresser and has a 10 pack-year history of cigarette smoking is scheduled for an annual physical examination. The nurse will plan to teach the patient about the increased risk for ________________

Give ketorolac (Toradol) 10 mg PO PRN for pain.

A 26-year-old patient with a history of polycystic kidney disease is admitted to the surgical unit after having knee surgery. Which of the routine postoperative orders is most important for the nurse to discuss with the health care provider?

Importance of genetic counseling

A 32-year-old patient is diagnosed with polycystic kidney disease. Which information is most appropriate for the nurse to include in teaching at this time?

Teach the patient how to perform Kegel exercises.

A 62-year-old asks the nurse for a perineal pad, stating that laughing or coughing causes leakage of urine. Which intervention is most appropriate to include in the care plan?

Costovertebral tenderness

A 72-year-old who has benign prostatic hyperplasia is admitted to the hospital with chills, fever, and vomiting. Which finding by the nurse will be most helpful in determining whether the patient has an upper urinary tract infection (UTI)?

Assist the patient to the bathroom every 2 hours during the day.

A 78-year-old who has been admitted to the hospital with dehydration is confused and incontinent of urine. Which nursing action will be best to include in the plan of care?

Large container for urine

A creatinine clearance test is ordered for a hospitalized patient with possible renal insufficiency. Which equipment will the nurse need to obtain?

Monitor the urine output after the procedure.

A hospitalized patient with a decreased glomerular filtration rate is scheduled to have an intravenous pyelogram (IVP). Which action will be included in the plan of care?

blood urea nitrogen (BUN) and creatinine.

A new order for IV gentamicin (Garamycin) 60 mg BID is received for a patient with diabetes who has pneumonia. When evaluating for adverse effects of the medication, the nurse will plan to monitor the patient's _______________

Which are characteristic clinical manifestations of acute poststreptococal glomerulonephritis (APSGN)? A. Hematuria and dependent edema B. Anterior pelvic pain and glycosuria C. Lower back pain and inflamed kidneys D. Dyspnea and dull, aching pain over the urinary bladder

A. Hematuria and dependent edema Generalized body edema, hypertension, oliguria, hematuria with a smoky or rusty appearance, and proteinuria may occur. Fluid retention results from decreased glomerular filtration. The edema appears initially in low-pressure tissues, such as around the eyes (periorbital edema), but it later progresses to involve the total body as ascites or peripheral edema in the legs.

What should discharge teaching for the patient with pyelonephritis include (select all that apply)? A. Rest often throughout the day B. Eliminate all juices from the diet C. Drink at least 8 glasses of fluid per day D. Complete the entire antibiotic prescription E. Return to the physician for a follow-up urine culture

A. Rest often throughout the day C. Drink at least 8 glasses of fluid per day D. Complete the entire antibiotic prescription E. Return to the physician for a follow-up urine culture Nursing interventions depend on the severity of symptoms. These interventions include teaching the patient about the disease process, with emphasis on (1) the need to continue drugs as prescribed, (2) the need for a follow-up urine culture to ensure proper management, and (3) identification of risk for recurrence or relapse. In addition to antibiotic therapy, encourage the patient to drink at least eight glasses of fluid every day, even after the infection has been treated. Rest is often indicated to increase patient comfort. There is no need to eliminate juice.

Which urinalysis results most likely indicate glomerular damage? A. Smoky color; 30 mg/dL of protein; pH of 6.2 B. Cloudy, yellow; 50 WBCs/hpf; pH of 8.2; numerous casts C. Cloudy, brown; ammonia odor; specific gravity of 1.030; 3 RBCs/hpf. D. Clear, colorless; trace of glucose; trace of ketones; osmolality of 500 mOsm/kg (500 mmol/kg)

A. Smoky color; 30 mg/dL of protein; pH of 6.2 The clinical manifestations of acute poststreptococcal glomerulonephritis (APSGN) appear as a variety of signs and symptoms, including generalized body edema, hypertension, oliguria, hematuria with a smoky or rusty appearance, and proteinuria may occur.

To help reduce the incidence of acute glomerulonephritis, public health teaching can stress the importance of A. early treatment of strep throat. B. obtaining a yearly screening urinalysis. C. proper urinary hygiene to prevent cystitis. D. early immunizations against streptococcal pneumonia.

A. early treatment of strep throat. One of the most important ways to prevent APSGN is to encourage early diagnosis and treatment of sore throats and skin lesions. If streptococci are found in the culture, treatment with appropriate antibiotic therapy (usually penicillin) is essential. The patient must be encouraged to take the full course of antibiotics to ensure that the bacteria have been eradicated.

serum phosphate.

Before administration of calcium carbonate (Caltrate) to a patient with chronic kidney disease (CKD), the nurse should check the laboratory value for _______________

The respiratory rate is 38 breaths/minute.

Following an intravenous pyelogram (IVP), all of the following assessment data are obtained. Which one requires immediate action by the nurse?

"I will clean the catheter carefully before and after each catheterization."

After the home health nurse teaches a patient with a neurogenic bladder how to use intermittent catheterization for bladder emptying, which patient statement indicates that the teaching has been effective?

Report the patient's symptoms to the health care provider.

After the insertion of an arteriovenous graft (AVG) in the right forearm, a patient complains of pain and coldness of the right fingers. Which action should the nurse take?

The caregiver of a child with a history of ear infections calls the nurse and reports that her son has just told her his urine "looks funny." He also complains of a headache, and his mother reports that his eyes are puffy. Although he had a fever 2 days ago, his temperature is now down to 100 degrees Fahrenheit. The nurse encourages the mother to have the child seen by the care provider because the nurse suspects the child may have a) Rheumatic fever b) A urinary tract infection c) Acute glomerulonephritis d) Lipoid nephrosis (idiopathic nephrotic syndrome)

Acute glomerulonephritis Correct Explanation: Acute glomerulonephritis is a condition that appears to be an allergic reaction to specific infections, most often group A beta-hemolytic streptococcal infections such as rheumatic fever. Presenting symptoms appear one to three weeks after the onset of a streptococcal infection such as strep throat, otitis media, tonsillitis, or impetigo. Usually the presenting symptom is grossly bloody urine. Periorbital edema may accompany or precede hematuria. Fever may be 103 to 104 degrees Fahrenheit at the onset but decreases in a few days to about 100 degrees Fahrenheit. Slight headache and malaise are usual, and vomiting may occur.

Check the chart for the most recent blood potassium level.

After noting lengthening QRS intervals in a patient with acute kidney injury (AKI), which action should the nurse take first?

bladder cancer.

After obtaining the health history for a 25-year-old who smokes two packs of cigarettes daily, the nurse will plan to do teaching about the increased risk for __________

"I should start taking a high potency multiple vitamin every morning."

After teaching a patient with interstitial cystitis about management of the condition, the nurse determines that further instruction is needed when the patient says, __________________

20. The nurse is assessing a client diagnosed with cystitis. To percuss the kidneys, the nurse locates the costovertebral angle, which is formed by the spinal column and rib number:

Amswer 12; RATIONALES: Kidney percussion is done to check for costovertebral angle tenderness that occurs with inflammation. Percussing over the kidneys is done with the client sitting down. The nurse should place the ball of her nondominant hand on the client's back at the costovertebral angle — the angle formed by the spinal column and the 12th rib. The nurse should then strike the ball of her hand with the ulnar surface of her other hand and percuss bilaterally.

Leave a light on in the bathroom during the night.

An 82-year-old man has been admitted with benign prostatic hyperplasia. Which of the following is most appropriate to include in the nursing plan of care?

Insert a urinary retention catheter.

An 88-year-old with benign prostatic hyperplasia (BPH) has a markedly distended bladder and is agitated and confused. Which of the following interventions prescribed by the health care provider should the nurse implement first?

63. The client is prescribed continuous bladder irrigation at a rate of 60 gtt/minute. The nurse hangs a 2 L bag of sterile solution with tubing on a three-legged I.V. pole. She then attaches the tubing to the client's three-way urinary catheter, adjusts the flow rate, and leaves the room. Which important procedural step did the nurse fail to follow? 1. Evaluating patency of the drainage lumen 2. Counter-balancing the I.V. pole 3. Attaching the infusion set to an infusion pump 4. Collecting a urine specimen before beginning irrigation

Answer 1 RATIONALES: The nurse should evaluate patency of the drainage tubing before leaving the client's room. If the lumen is obstructed, the solution infuses into the bladder but isn't eliminated through the drainage tubing, a situation that may cause client injury. Balancing the pole is important; however, the nurse would have had to address this issue immediately after hanging the 2 L bag. Using an I.V. pump isn't necessary for continuous bladder irrigation. Unless specifically ordered, obtaining a urine specimen before beginning continuous bladder irrigation isn't necessary.

50. A client is scheduled for a renal clearance test. The nurse should explain that this test is done to assess the kidneys' ability to remove a substance from the plasma in: 1. 1 minute. 2. 30 minutes. 3. 1 hour. 4. 24 hours.

Answer 1 RATIONALES: The renal clearance test determines the kidneys' ability to remove a substance from the plasma in 1 minute. It doesn't measure the kidneys' ability to remove a substance over a longer period.

96. A client reports experiencing vulvar pruritus. Which assessment factor may indicate that the client has an infection caused by Candida albicans? 1. Cottage cheese-like discharge 2. Yellow-green discharge 3. Gray-white discharge 4. Discharge with a fishy odor

Answer 1 RATIONALES: The symptoms of C. albicans include itching and a scant white discharge that has the consistency of cottage cheese. Yellow-green discharge is a sign of Trichomonas vaginalis. Gray-white discharge and a fishy odor are signs of Gardnerella vaginal

87. A client comes to the emergency department complaining of severe pain in the right flank, nausea, and vomiting. The physician tentatively diagnoses right ureterolithiasis (renal calculi). When planning this client's care, the nurse should assign highest priority to which nursing diagnosis? 1. Acute pain 2. Risk for infection 3. Impaired urinary elimination 4. Imbalanced nutrition: Less than body requirements

Answer 1 RATIONALES: Ureterolithiasis typically causes such acute, severe pain that the client can't rest and becomes increasingly anxious. Therefore, the nursing diagnosis of Acute pain takes highest priority. Diagnoses of Risk for infection and Impaired urinary elimination are appropriate when the client's pain is controlled. A diagnosis of Imbalanced nutrition: Less than body requirements isn't pertinent at this time.

84. A triple-lumen indwelling urinary catheter is inserted for continuous bladder irrigation following a transurethral resection of the prostate. In addition to balloon inflation, the functions of the three lumens include: 1. continuous inflow and outflow of irrigation solution. 2. intermittent inflow and continuous outflow of irrigation solution. 3. continuous inflow and intermittent outflow of irrigation solution. 4. intermittent flow of irrigation solution and prevention of hemorrhage.

Answer 1 RATIONALES: When preparing for continuous bladder irrigation, a triple-lumen indwelling urinary catheter is inserted. The three lumens provide for balloon inflation and continuous inflow and outflow of irrigation solution.

10. The nurse is caring for a client with chronic renal failure. The laboratory results indicate hypocalcemia and hyperphosphatemia. When assessing the client, the nurse should be alert for which of the following? 1. Trousseau's sign 2. Cardiac arrhythmias 3. Constipation 4. Decreased clotting time 5. Drowsiness and lethargy 6. Fractures

Answer 1,2,6 RATIONALES: Hypocalcemia is a calcium deficit that causes nerve fiber irritability and repetitive muscle spasms. Signs and symptoms of hypocalcemia include Trousseau's sign, cardiac arrhythmias, diarrhea, increased clotting times, anxiety, and irritability. The calcium-phosphorus imbalance leads to brittle bones and pathologic fractures.

58. The nurse is planning to administer a sodium polystyrene sulfonate (Kayexalate) enema to a client with a potassium level of 6.2 mEq/L. Correct administration and the effects of this enema would include having the client: 1. retain the enema for 30 minutes to allow for sodium exchange; afterward, the client should have diarrhea. 2. retain the enema for 30 minutes to allow for glucose exchange; afterward, the client should have diarrhea. 3. retain the enema for 60 minutes to allow for sodium exchange; diarrhea isn't necessary to reduce the potassium level. 4. retain the enema for 60 minutes to allow for glucose exchange; diarrhea isn't necessary to reduce the potassium level.

Answer 1: RATIONALES: Kayexalate is a sodium-exchange resin. Thus, the client will gain sodium as potassium is lost in the bowel. For the exchange to occur, Kayexalate must be in contact with the bowel for at least 30 minutes. Sorbitol in the Kayexalate enema causes diarrhea, which increases potassium loss and decreases the potential for Kayexalate retention.

5. A client returns to the medical-surgical unit after coronary artery bypass graft surgery, which was complicated by prolonged cardiopulmonary bypass and hypotension. After 3 hours in the unit, the client's condition stabilizes. However, the urine output has decreased despite adequate filling pressures. The nurse expects the physician to add which drug, at which flow rate, to the client's regimen? 1. Dopamine (Intropin), 3 mcg/kg/min 2. Epinephrine, 2 mcg/kg/min 3. Dopamine (Intropin), 8 mcg/kg/min 4. Epinephrine, 4 mcg/kg/min

Answer 1: Answer 1: RATIONALES: This client is at high risk for acute prerenal failure secondary to decreased renal perfusion during surgery. To dilate the renal arteries and help prevent renal shutdown, the physician is likely to prescribe dopamine at a low flow rate (2 to 5 mcg/kg/min). Although this drug has mixed dopaminergic and beta activity when given at 5 to 10 mcg/kg/min, the client is stabilized and thus doesn't need the beta effects from the higher flow rate — or the sympathomimetic effects of epinephrine. The dopaminergic effects of dopamine increase renal perfusion, contractility, and vasodilation. Stimulation of beta receptors causes beta effects — namely, increases in the heart rate, myocardial contraction force, and cardiac conduction

51. After undergoing a total cystectomy and urinary diversion, a client has a Kock pouch (continent internal reservoir). Which statement by the client indicates a need for further teaching? 1. "I'll have to wear an external collection pouch for the rest of my life." 2. "I should eat foods from all the food groups." 3. "I'll need to drink at least eight glasses of water a day." 4. "I'll have to catheterize my pouch every 2 hours."

Answer 1: RATIONALES: An internal collection pouch, such as the Kock pouch, allows the client to perform self-catheterization for ileal drainage. This pouch is an internal reservoir, eliminating the need for an external collection pouch. A well-balanced diet is essential for healing; the client need not include or exclude particular foods. The client should drink at least eight glasses of fluid daily to prevent calculi formation and urinary tract infection (UTI). Intervals between pouch drainings should be increased gradually until the pouch is emptied two to four times daily.

77. A client develops acute renal failure (ARF) after receiving I.V. therapy with a nephrotoxic antibiotic. Because the client's 24-hour urine output totals 240 ml, the nurse suspects that the client is at risk for: 1. cardiac arrhythmia. 2. paresthesia. 3. dehydration. 4. pruritus.

Answer 1: RATIONALES: As urine output decreases, the serum potassium level rises; if it rises sufficiently, hyperkalemia may occur, possibly triggering a cardiac arrhythmia. Hyperkalemia doesn't cause paresthesia (sensations of numbness and tingling). Dehydration doesn't occur during this oliguric phase of ARF, although typically it does arise during the diuretic phase. In a client with ARF, pruritus results from increased phosphates and isn't associated with hyperkalemia.

When caring for a client with acute renal failure (ARF), the nurse expects to adjust the dosage or dosing schedule of certain drugs. Which of the following drugs would not require such adjustment? 1. acetaminophen (Tylenol) 2. gentamicin sulfate (Garamycin) 3. cyclosporine (Sandimmune) 4. ticarcillin disodium (Ticar)

Answer 1: RATIONALES: Because acetaminophen is metabolized in the liver, its dosage and dosing schedule need not be adjusted for a client with ARF. In contrast, the dosages and schedules for gentamicin and ticarcillin, which are metabolized and excreted by the kidney, should be adjusted. Because cyclosporine may cause nephrotoxicity, the nurse must monitor both the dosage and blood drug level in a client receiving this drug.

15. A 75-year-old client with renal insufficiency is admitted to the hospital with pneumonia. He's being treated with gentamicin (Garamycin), which can be nephrotoxic. Which laboratory value should be closely monitored? 1. Blood urea nitrogen 2. Sodium level 3. Alkaline phosphatase 4. White blood cell (WBC) count

Answer 1: RATIONALES: Blood urea nitrogen and creatinine levels should be closely monitored to detect elevations caused by nephrotoxicity. Sodium level should be routinely monitored in all hospitalized clients. Alkaline phosphatase helps evaluate liver function. The WBC count should be monitored to evaluate the effectiveness of the antibiotic; it doesn't help evaluate kidney function.

95. The nurse is caring for a client who had a stroke. Which nursing intervention promotes urinary continence? 1. Encouraging intake of at least 2 L of fluid daily 2. Giving the client a glass of soda before bedtime 3. Taking the client to the bathroom twice per day 4. Consulting with a dietitian

Answer 1: RATIONALES: By encouraging a daily fluid intake of at least 2 L, the nurse helps fill the client's bladder, thereby promoting bladder retraining by stimulating the urge to void. The nurse shouldn't give the client soda before bedtime; soda acts as a diuretic and may make the client incontinent. The nurse should take the client to the bathroom or offer the bedpan at least every 2 hours throughout the day; twice per day is insufficient. Consultation with a dietitian won't address the problem of urinary incontinence.

14. A client with acute renal failure is undergoing dialysis for the first time. The nurse monitors the client closely for dialysis equilibrium syndrome, a complication that is most common during the first few dialysis sessions. Typically, dialysis equilibrium syndrome causes: 1. confusion, headache, and seizures. 2. acute bone pain and confusion. 3. weakness, tingling, and cardiac arrhythmias. 4. hypotension, tachycardia, and tachypnea.

Answer 1: RATIONALES: Dialysis equilibrium syndrome causes confusion, a decreasing level of consciousness, headache, and seizures. These findings, which may last several days, probably result from a relative excess of interstitial or intracellular solutes caused by rapid solute removal from the blood. The resultant organ swelling interferes with normal physiological functions. To prevent this syndrome, many dialysis centers keep first-time sessions short and use a reduced blood flow rate. Acute bone pain and confusion are associated with aluminum intoxication, another potential complication of dialysis. Weakness, tingling, and cardiac arrhythmias suggest hyperkalemia, which is associated with renal failure. Hypotension, tachycardia, and tachypnea signal hemorrhage, another dialysis complication.

18 A client is admitted for treatment of glomerulonephritis. On initial assessment, the nurse detects one of the classic signs of acute glomerulonephritis of sudden onset. Such signs include: 1. generalized edema, especially of the face and periorbital area. 2. green-tinged urine. 3. moderate to severe hypotension. 4. polyuria.

Answer 1: RATIONALES: Generalized edema, especially of the face and periorbital area, is a classic sign of acute glomerulonephritis of sudden onset. Other classic signs and symptoms of this disorder include hematuria (not green-tinged urine), proteinuria, fever, chills, weakness, pallor, anorexia, nausea, and vomiting. The client also may have moderate to severe hypertension (not hypotension), oliguria or anuria (not polyuria), headache, reduced visual acuity, and abdominal or flank pain.

29. A client with benign prostatic hyperplasia doesn't respond to medical treatment and is admitted to the facility for prostate gland removal. Before providing preoperative and postoperative instructions to the client, the nurse asks the surgeon which prostatectomy procedure will be done. What is the most widely used procedure for prostate gland removal? 1. Transurethral resection of the prostate (TURP) 2. Suprapubic prostatectomy 3. Retropubic prostatectomy 4. Transurethral laser incision of the prostate

Answer 1: RATIONALES: TURP is the most widely used procedure for prostate gland removal. Because it requires no incision, TURP is especially suitable for men with relatively minor prostatic enlargements and for those who are poor surgical risks. Suprapubic prostatectomy, retropubic prostatectomy, and transurethral laser incision of the prostate are less common procedures; they all require an incision.

56. A client with a genitourinary problem is being examined in the emergency department. When palpating the client's kidneys, the nurse should keep in mind which anatomic fact? 1. The left kidney usually is slightly higher than the right one. 2. The kidneys are situated just above the adrenal glands. 3. The average kidney is approximately 5 cm (2″) long and 2 to 3 cm (¾″ to 1-1/8″) wide. 4. The kidneys lie between the 10th and 12th thoracic vertebrae.

Answer 1: RATIONALES: The left kidney usually is slightly higher than the right one. An adrenal gland lies atop each kidney. The average kidney measures approximately 11 cm (4-3/8″) long, 5 to 5.8 cm (2″ to 2¼″) wide, and 2.5 cm (1″) thick. The kidneys are located retroperitoneally, in the posterior aspect of the abdomen, on either side of the vertebral column. They lie between the 12th thoracic and 3rd lumbar vertebrae.

85. A client comes to the emergency department complaining of sudden onset of sharp, severe pain in the lumbar region, which radiates around the side and toward the bladder. The client also reports nausea and vomiting and appears pale, diaphoretic, and anxious. The physician tentatively diagnoses renal calculi and orders flat-plate abdominal X-rays. Renal calculi can form anywhere in the urinary tract. What is their most common formation site? 1. Kidney 2. Ureter 3. Bladder 4. Urethra

Answer 1: RATIONALES: The most common site of renal calculi formation is the kidney. Calculi may travel down the urinary tract with or without causing damage and may lodge anywhere along the tract or may stay within the kidney. The ureter, bladder, and urethra are less common sites of renal calculi formation.

75. Two weeks after being diagnosed with a streptococcal infection, a client develops fatigue, a low-grade fever, and shortness of breath. The nurse auscultates bilateral crackles and observes neck vein distention. Urinalysis reveals red and white blood cells and protein. After the physician diagnoses poststreptococcal glomerulonephritis, the client is admitted to the medical-surgical unit. Which immediate action should the nurse take? 1. Place the client on bed rest. 2. Provide a high-protein, fluid-restricted diet. 3. Prepare to assist with insertion of a Tenckhoff catheter for hemodialysis. 4. Place the client on a sheepskin, and monitor for increasing edema.

Answer 1: RATIONALES: The nurse immediately must enforce bed rest for a client with glomerulonephritis to ensure a complete recovery and help prevent complications. Depending on disease severity, the client may require fluid, sodium, potassium, and protein restrictions. Because of the risk of altered urinary elimination related to oliguria, this client may require hemodialysis or plasmapheresis for several weeks until renal function improves; however, a Tenckhoff catheter is used in peritoneal dialysis, not hemodialysis. Although comfort measures such as placing the client on a sheepskin are important, they don't take precedence.

88. After undergoing renal arteriogram, in which the left groin was accessed, the client complains of left calf pain. Which intervention should the nurse perform first? 1. Assess peripheral pulses in the left leg. 2. Place cool compresses on the calf. 3. Exercise the leg and foot. 4. Assess for anaphylaxis.

Answer 1: RATIONALES: The nurse should begin by assessing peripheral pulses in the left leg to determine if blood flow was interrupted by the procedure. The client may also have thrombophlebitis. Cool compresses aren't used to relieve pain and inflammation in thrombophlebitis. The leg should remain straight after the procedure. Calf pain isn't a symptom of anaphylaxis.

9. The nurse is caring for a 25-year-old female client diagnosed with a chlamydia infection. The nurse teaches the client about disease transmission and advises the client to inform her sexual partners of the infection. The client refuses, stating, "This is my business and I'm not telling anyone. Beside, chlamydia doesn't cause any harm like the other STDs." How should the nurse proceed? 1. Educate the client about why it's important to inform sexual contacts so they can receive treatment. 2. Inform the health department that this client contracted a sexually transmitted disease. 3. Inform the client's sexual contacts of their possible exposure to chlamydia. 4. Do nothing because the client's sexual habits place her at risk for contracting other sexually transmitted diseases.

Answer 1: RATIONALES: The nurse should educate the client about the disease and how it impacts a person's health. Further education allows the client to make an informed decision about notifying sexual contacts. The nurse must maintain client confidentiality unless law mandates reporting the illness; contacting sexual contacts breeches client confidentiality. Option 4 is judgmental; everyone is entitled to health care regardless of their health habits.

86. A client is scheduled to undergo surgical creation of an ileal conduit. The primary nurse educates the client about surgery and the postoperative period. The nurse informs the client that many members of the health care team (including a mental health practitioner) will see him. A mental health practitioner should be involved in the client's care to: 1. assess whether the client is a good candidate for surgery. 2. help the client cope with the anxiety associated with changes in body image. 3. assess suicidal risk postoperatively. 4. evaluate the client's need for mental health intervention.

Answer 2 RATIONALES: Many clients who undergo surgery for creation of an ileal conduit experience anxiety associated with changes in body image. The mental health practitioner can help with client cope these feelings of anxiety. Mental health practitioners don't evaluate whether the client is a surgical candidate. None of the evidence suggests that urinary diversion surgery, such as creation of an ileal conduit, places the client at risk for suicide. Although evaluating the need for mental health intervention is always important, this client displays no behavioral changes that suggest intervention is necessary at this time.

65. A client receiving total parental nutrition is prescribed a 24-hour urine test. When initiating a 24-hour urine specimen, the collection time should: 1. start with the first voiding. 2. start after a known voiding. 3. always be with the first morning urine. 4. always be the evening's last void as the last sample.

Answer 2 RATIONALES: When initiating a 24-hour urine specimen, have the client void, then start the timing. The collection should start on an empty bladder. The exact time the test starts isn't important but it's commonly started in the morning.

8. A client with renal dysfunction of acute onset comes to the emergency department complaining of fatigue, oliguria, and coffee-colored urine. When obtaining the client's history to check for significant findings, the nurse should ask about: 1. chronic, excessive acetaminophen use. 2. recent streptococcal infection. 3. childhood asthma. 4. family history of pernicious anemia.

Answer 2: RATIONALES: A skin or upper respiratory infection of streptococcal origin may lead to acute glomerulonephritis. Other infections that may be linked to renal dysfunction include infectious mononucleosis, mumps, measles, and cytomegalovirus. Chronic, excessive acetaminophen use isn't nephrotoxic, although it may be hepatotoxic. Childhood asthma and a family history of pernicious anemia aren't significant history findings for a client with renal dysfunction.

93. Which conditions are functions of antidiuretic hormone (ADH)? 1. Sodium absorption and potassium excretion 2. Water reabsorption and urine concentration 3. Water reabsorption and urine dilution 4. Sodium reabsorption and potassium retention

Answer 2: RATIONALES: ADH stimulates the renal tubules to reabsorb water, thereby concentrating urine. Aldosterone is responsible for sodium reabsorption and potassium excretion by the kidneys.

67. A client is admitted with a diagnosis of acute renal failure. The nurse should monitor closely for: 1. enuresis. 2. drug toxicity. 3. lethargy. 4. insomnia.

Answer 2: RATIONALES: Acute renal failure is characterized by oliguria and rapid accumulation of nitrogen waste in the blood. Kidneys excrete medications, so the nurse should monitor the client closely for drug toxicity. With decreased urinary output or no output, enuresis shouldn't occur. The client will most likely feel lethargic, but this isn't as serious a problem as drug toxicity. The client isn't likely to have insomnia, but, may instead want to sleep most of the time.

38. Because of difficulties with hemodialysis, peritoneal dialysis is initiated to treat a client's uremia. Which finding signals a significant problem during this procedure? 1. Blood glucose level of 200 mg/dl 2. White blood cell (WBC) count of 20,000/mm3 3. Potassium level of 3.5 mEq/L 4. Hematocrit (HCT) of 35%

Answer 2: RATIONALES: An increased WBC count indicates infection, probably resulting from peritonitis, which may have been caused by insertion of the peritoneal catheter into the peritoneal cavity. Peritonitis can cause the peritoneal membrane to lose its ability to filter solutes; therefore, peritoneal dialysis would no longer be a treatment option for this client. Hyperglycemia occurs during peritoneal dialysis because of the high glucose content of the dialysate; it's readily treatable with sliding-scale insulin. A potassium level of 3.5 mEq/L can be treated by adding potassium to the dialysate solution. An HCT of 35% is lower than normal. However, in this client, the value isn't abnormally low because of the daily blood samplings. A lower HCT is common in clients with chronic renal failure because of the lack of erythropoietin.

57. A client with chronic renal failure (CRF) is receiving a hemodialysis treatment. After hemodialysis, the nurse knows that the client is most likely to experience: 1. hematuria. 2. weight loss. 3. increased urine output. 4. increased blood pressure.

Answer 2: RATIONALES: Because CRF causes loss of renal function, the client with this disorder retains fluid. Hemodialysis removes this fluid, causing weight loss. Hematuria is unlikely to follow hemodialysis because the client with CRF usually forms little or no urine. Hemodialysis doesn't increase urine output because it doesn't correct the loss of kidney function, which severely decreases urine production in this disorder. By removing fluids, hemodialysis decreases rather than increases the blood pressure.

28. A client with chronic renal failure who receives hemodialysis three times weekly has a hemoglobin (Hb) level of 7 g/dl. The most therapeutic pharmacologic intervention would be to administer: 1. ferrous sulfate (Feratab). 2. epoetin alfa (Epogen) 3. filgrastim (Neupogen) 4. enoxaparin (Lovenox)

Answer 2: RATIONALES: Chronic renal failure diminishes the production of erythropoietin by the kidneys and leads to a subnormal Hb level. (Normal Hb level is 13 to 18 g/dl in men and 12 to 16 g/dl in women.) An effective pharmacologic treatment for this is epoetin alfa, a recombinant erythropoietin. Because the client's anemia is caused by a deficiency of erythropoietin and not a deficiency of iron, administering ferrous sulfate would be ineffective. Neither filgrastim, a drug used to stimulate neutrophils, nor enoxaparin (low-molecular-weight heparin) will raise the client's Hb level.

43. A client with suspected renal dysfunction is scheduled for excretory urography. The nurse reviews the history for conditions that may warrant changes in client preparation. Normally, a client should be mildly hypovolemic (fluid depleted) before excretory urography. Which history finding would call for the client to be well hydrated instead? 1. Cystic fibrosis 2. Multiple myeloma 3. Gout 4. Myasthenia gravis

Answer 2: RATIONALES: Fluid depletion before excretory urography is contraindicated in clients with multiple myeloma, severe diabetes mellitus, and uric acid nephropathy — conditions that can seriously compromise renal function in fluid-depleted clients with reduced renal perfusion. If these clients must undergo excretory urography, they should be well hydrated before the test. Cystic fibrosis, gout, and myasthenia gravis don't necessitate changes in client preparation for excretory urography.

31. The physician prescribes norfloxacin (Noroxin), for a client with a urinary tract infection (UTI). The client asks the nurse how long to continue taking the drug. For an uncomplicated UTI, the usual duration of norfloxacin therapy is: 1. 3 to 5 days. 2. 7 to 10 days. 3. 12 to 14 days. 4. 10 to 21 days.

Answer 2: RATIONALES: For an uncomplicated UTI, norfloxacin therapy usually lasts 7 to 10 days. Taking the drug for less than 7 days wouldn't eradicate such an infection. Taking it for more than 10 days isn't necessary. Only a client with a complicated UTI must take norfloxacin for 10 to 21 days.

74. A client comes to the outpatient department complaining of vaginal discharge, dysuria, and genital irritation. Suspecting a sexually transmitted disease (STD), the physician orders diagnostic testing of the vaginal discharge. Which STD must be reported to the public health department? 1. Chlamydia 2. Gonorrhea 3. Genital herpes 4. Human papillomavirus infection

Answer 2: RATIONALES: Gonorrhea must be reported to the public health department. Chlamydia, genital herpes, and human papillomavirus infection aren't reportable diseases.

76. A client with bladder cancer has had the bladder removed and an ileal conduit created for urine diversion. While changing this client's pouch, the nurse observes that the area around the stoma is red, weeping, and painful. What should the nurse conclude? 1. The skin wasn't lubricated before the pouch was applied. 2. The pouch faceplate doesn't fit the stoma. 3. A skin barrier was applied properly. 4. Stoma dilation wasn't performed.

Answer 2: RATIONALES: If the pouch faceplate doesn't fit the stoma properly, the skin around the stoma will be exposed to continuous urine flow from the stoma, causing excoriation and red, weeping, and painful skin. A lubricant shouldn't be used because it would prevent the pouch from adhering to the skin. When properly applied, a skin barrier prevents skin excoriation. Stoma dilation isn't performed with an ileal conduit, although it may be done with a colostomy if ordered.

90. A client undergoes extracorporeal shock wave lithotripsy. Before discharge, the nurse should provide which instruction? 1. "Take your temperature every 4 hours." 2. "Increase your fluid intake to 2 to 3 L per day." 3. "Apply an antibacterial dressing to the incision daily." 4. "Be aware that your urine will be cherry-red for 5 to 7 days."

Answer 2: RATIONALES: Increasing fluid intake flushes the renal calculi fragments through — and prevents obstruction of — the urinary system. Measuring temperature every 4 hours isn't needed. Lithotripsy doesn't require an incision. Hematuria may occur for a few hours after lithotripsy but should then disappear.

100. The physician enters a computer order for the nurse to irrigate a client's nephrostomy tube every four hours to maintain patency. The nurse irrigates the tube using sterile technique. After irrigating the tube, the nurse decides that she can safely use the same irrigation set for her 8-hour shift if she covers the set with a paper, sterile drape. This action by the nurse is: 1. appropriate because the irrigation just checks for patency. 2. inappropriate because irrigation requires strict sterile technique. 3. appropriate because the irrigation set will only be used during an 8-hour period. 4. inappropriate because the sterile drape must be cloth, not paper.

Answer 2: RATIONALES: Irritating a nephrostomy tube requires strict sterile technique; therefore, reusing the irrigation set (even if covered by a sterile drape) is inappropriate. Bacteria can proliferate inside the syringe and irrigation container. Although this procedure checks patency, it requires sterile technique to prevent the introduction of bacteria into the kidney. The material in which the sterile drape is made is irrelevant because a sterile drape doesn't deter bacterial growth in the irrigation equipment

46. A client with an indwelling urinary catheter is suspected of having a urinary tract infection. The nurse should collect a urine specimen for culture and sensitivity by: 1. disconnecting the tubing from the urinary catheter and letting the urine flow into a sterile container. 2. wiping the self-sealing aspiration port with antiseptic solution and aspirating urine with a sterile needle. 3. draining urine from the drainage bag into a sterile container. 4. clamping the tubing for 60 minutes and inserting a sterile needle into the tubing above the clamp to aspirate urine.

Answer 2: RATIONALES: Most catheters have a self-sealing port for obtaining a urine specimen. Antiseptic solution is used to reduce the risk of introducing microorganisms into the catheter. Tubing shouldn't be disconnected from the urinary catheter. Any break in the closed urine drainage system may allow the entry of microorganisms. Urine in urine drainage bags may not be fresh and may contain bacteria, giving false test results. When there is no urine in the tubing, the catheter may be clamped for no more than 30 minutes to allow urine to collect.

32. The nurse is reviewing the report of a client's routine urinalysis. Which value should the nurse consider abnormal? 1. Specific gravity of 1.03 2. Urine pH of 3.0 3. Absence of protein 4. Absence of glucose

Answer 2: RATIONALES: Normal urine pH is 4.5 to 8; therefore, a urine pH of 3.0 is abnormal. Urine specific gravity normally ranges from 1.002 to 1.035, making this client's value normal. Normally, urine contains no protein, glucose, ketones, bilirubin, bacteria, casts, or crystals. Red blood cells should measure 0 to 3 per high-power field; white blood cells, 0 to 4 per high-power field. Urine should be clear, with color ranging from pale yellow to deep amber.

97. The nurse is reviewing a client's fluid intake and output record. Fluid intake and urine output should relate in which way? 1. Fluid intake should be double the urine output. 2. Fluid intake should be approximately equal to the urine output. 3. Fluid intake should be half the urine output. 4. Fluid intake should be inversely proportional to the urine output.

Answer 2: RATIONALES: Normally, fluid intake is approximately equal to the urine output. Any other relationship signals an abnormality. For example, fluid intake that is double the urine output indicates fluid retention; fluid intake that is half the urine output indicates dehydration. Normally, fluid intake isn't inversely proportional to the urine output.

48. After having transurethral resection of the prostate (TURP), a client returns to the unit with a three-way indwelling urinary catheter and continuous closed bladder irrigation. Which finding suggests that the client's catheter is occluded? 1. The urine in the drainage bag appears red to pink. 2. The client reports bladder spasms and the urge to void. 3. The normal saline irrigant is infusing at a rate of 50 drops/min. 4. About 1,000 ml of irrigant have been instilled; 1,200 ml of drainage have been returned.

Answer 2: RATIONALES: Reports of bladder spasms and the urge to void suggest that a blood clot may be occluding the catheter. After TURP, urine normally appears red to pink, and normal saline irrigant usually is infused at a rate of 40 to 60 drops/min or according to facility protocol. The amount of returned fluid (1,200 ml) should correspond to the amount of instilled fluid, plus the client's urine output (1,000 ml + 200 ml), which reflects catheter patency.

19. The nurse suspects that a client with a temperature of 103.6° F (39.8° C) and an elevated white blood cell count is in the initial stage of sepsis. What is the most common cause of sepsis in hospitalized clients? 1. Respiratory infection 2. Urinary tract infection (UTI) 3. Vasculitis 4. Osteomyelitis

Answer 2: RATIONALES: Sepsis most commonly results from a UTI caused by gram-negative bacteria. Other causes of sepsis include infections of the biliary, GI, and gynecologic tracts. Respiratory infection, vasculitis, and osteomyelitis rarely cause sepsis in hospitalized clients.

34. A client who has cervical cancer is scheduled to undergo internal radiation. In teaching the client about the procedure, the nurse would be most accurate in telling the client: 1. she will be in a private room with unrestricted activities. 2. a bowel-cleansing procedure will precede radioactive implantation. 3. she will be expected to use a bedpan for urination. 4. the preferred position in bed will be semi-Fowler's.

Answer 2: RATIONALES: The client will receive an enema before the procedure because bowel motility during cervical radiation implant therapy can disrupt or dislodge the implants. The client will be in a private room, and activities will be restricted in order to keep the implants in place. To keep the bladder empty, an indwelling catheter will be used. Positioning in bed shouldn't exceed a 20-degree elevation because sitting up can cause the implants to move from their intended locations. Semi-Fowler's position is 45 degrees.

44. An 85-year-old client is transferred from a local assisted living center to the emergency department with depression and behavioral changes. The nurse notes that the client cries out when she approaches. When the nurse gains the client's confidence and performs an assessment, the nurse notes bruising of the labia and a lateral laceration in the perineal area. When the nurse asks the client about the injury, the client shakes her head and begins to cry "don't tell, don't tell." The nurse suspects sexual abuse. How should the nurse proceed? 1. Notify the physician of her findings immediately. 2. Attend to the client's physiological needs. 3. Notify the client's family. 4. Notify the rape crisis team.

Answer 2: RATIONALES: The nurse should attend to the client's immediate physiological needs including physical safety. Next, the nurse can notify the physician and the rape crisis team. The family should be notified if the client consents, but not until the rape investigation is complete.

53. When a client with an indwelling urinary catheter insists on walking to the hospital lobby to visit with family members, the nurse teaches him how to do this without compromising the catheter. Which client action indicates an accurate understanding of this information? 1. The client sets the drainage bag on the floor while sitting down. 2. The client keeps the drainage bag below the bladder at all times. 3. The client clamps the catheter drainage tubing while visiting with the family. 4. The client loops the drainage tubing below its point of entry into the drainage bag.

Answer 2: RATIONALES: To maintain effective drainage, the client should keep the drainage bag below the bladder; this allows the urine to flow by gravity from the bladder to the drainage bag. The client shouldn't lay the drainage bag on the floor because it could become grossly contaminated. The client shouldn't clamp the catheter drainage tubing because this impedes the flow of urine. To promote drainage, the client may loop the drainage tubing above — not below — its point of entry into the drainage bag.

80. Which statement describing urinary incontinence in the elderly is true? 1. Urinary incontinence is a normal part of aging. 2. Urinary incontinence isn't a disease. 3. Urinary incontinence in the elderly can't be treated. 4. Urinary incontinence is a disease.

Answer 2: RATIONALES: Urinary incontinence isn't a normal part of aging nor is it a disease. It may be caused by confusion, dehydration, fecal impaction, restricted mobility, or other causes. Certain medications, including diuretics, hypnotics, sedatives, anticholinergics, and antihypertensives, may trigger urinary incontinence. Most clients with urinary incontinence can be treated; some can be cured.

47. The nurse is caring for a male client with gonorrhea. The client asks how he can reduce his risk of contracting another sexually transmitted disease (STD). The nurse should instruct the client to: 1. ask all potential sexual partners if they have a sexually transmitted disease. 2. wear a condom every time he has intercourse. 3. consider intercourse safe if his partner has no visible discharge, lesions, or rashes. 4. expect to limit the number of sexual partners to less than five over his lifetime.

Answer 2: RATIONALES: Wearing a condom during intercourse considerably reduces the risk of contracting STDs. The other options may help reduce the risk of contracting an STD but not to the extent wearing a condom will. A monogamous relationship also reduces the risk of contracting STDs.

26. The nurse is inserting a urinary catheter into a client who is extremely anxious about the procedure. The nurse can facilitate the insertion by asking the client to: 1. initiate a stream of urine. 2. breathe deeply. 3. turn to the side. 4. hold the labia or shaft of the penis.

Answer 2: RATIONALES: When inserting a urinary catheter, facilitate insertion by asking the client to breathe deeply. Doing this will relax the urinary sphincter. Initiating a stream of urine isn't recommended during catheter insertion. Turning to the side or holding the labia or penis won't ease insertion, and doing so may contaminate the sterile field.

16. A client with a urinary tract infection is prescribed co-trimoxazole (trimethoprim-sulfamethoxazole). The nurse should provide which medication instruction? 1. "Take the medication with food." 2. "Drink at least eight 8-oz glasses of fluid daily." 3. "Avoid taking antacids during co-trimoxazole therapy." 4. "Don't be afraid to go out in the sun."

Answer 2: RATIONALES: When receiving a sulfonamide such as co-trimoxazole, the client should drink at least eight 8-oz glasses of fluid daily to maintain a urine output of at least 1,500 ml/day. Otherwise, inadequate urine output may lead to crystalluria or tubular deposits. For maximum absorption, the client should take this drug at least 1 hour before or 2 hours after meals. No evidence indicates that antacids interfere with the effects of sulfonamides. To prevent a photosensitivity reaction, the client should avoid direct sunlight during co-trimoxazole therapy.

68. During rectal examination, which finding would be further evidence of a urethral injury? 1. A low-riding prostate 2. The presence of a boggy mass 3. Absent sphincter tone 4. A positive Hemoccult

Answer 2: RATIONALES: When the urethra is ruptured, a hematoma or collection of blood separates the two sections of urethra. This may feel like a boggy mass on rectal examination. Because of the rupture and hematoma, the prostate becomes high riding. A palpable prostate gland usually indicates a nonurethral injury. Absent sphincter tone would refer to a spinal cord injury. The presence of blood would probably correlate with GI bleeding or a colon injury.

99. Which clinical finding would the nurse look for in a client with chronic renal failure? 1. Hypotension 2. Uremia 3. Metabolic alkalosis 4. Polycythemia

Answer 2; RATIONALES: Uremia is the buildup of nitrogenous wastes in the blood, evidenced by an elevated blood urea nitrogen and creatine levels. Uremia, anemia, and acidosis are consistent clinical manifestations of chronic renal failure. Metabolic acidosis results from the inability to excrete hydrogen ions. Anemia results from a lack of erythropoietin. Hypertension (from fluid overload) may or may not be present in chronic renal failure. Hypotension, metabolic alkalosis, and polycythemia aren't present in renal failure.

41. Which of the following is an appropriate nursing diagnosis for a client with renal calculi? 1. Ineffective renal tissue perfusion 2. Functional urinary incontinence 3. Risk for infection 4. Decreased cardiac output

Answer 3 RATIONALES: Infection can occur with renal calculi from urine stasis caused by obstruction. Options 1 and 4 aren't appropriate for this client, and retention of urine, rather than incontinence, usually occurs.

91. After trying to conceive for a year, a couple consults an infertility specialist. When obtaining a history from the husband, the nurse inquires about childhood infectious diseases. Which childhood infectious disease most significantly affects male fertility? 1. Chickenpox 2. Measles 3. Mumps 4. Scarlet fever

Answer 3 RATIONALES: Mumps is the childhood infectious disease that most significantly affects male fertility. Chickenpox, measles, and scarlet fever don't affect male fertility.

1. The nurse is caring for a male client with gonorrhea who's receiving ceftriaxone and doxycycline. The client asks the nurse why he's receiving two antibiotics. How should the nurse respond? 1. "Because there are many resistant strains of gonorrhea, more than one antibiotic may be required for successful treatment." 2. "The combination of these two antibiotics reduces the risk of reinfection." 3. "Many people infected with gonorrhea are infected with chlamydia as well." 4. "This combination of medications will eradicate the infection faster than a single antibiotic."

Answer 3: Answer 3: RATIONALES: Treatment for gonorrhea includes the antibiotic ceftriaxone. Because many people with gonorrhea have a coexisting chlamydial infection, doxycycline or azithromycin is prescribed as well. There has been an increase in the number of resistant strains of gonorrhea, but that isn't the reason for this dual therapy. This combination of antibiotics doesn't reduce the risk of reinfection or provide a faster cure.

25. A client in the short-procedure unit is recovering from renal angiography in which a femoral puncture site was used. When providing postprocedure care, the nurse should: 1. keep the client's knee on the affected side bent for 6 hours. 2. apply pressure to the puncture site for 30 minutes. 3. check the client's pedal pulses frequently. 4. remove the dressing on the puncture site after vital signs stabilize.

Answer 3: RATIONALES: After renal angiography involving a femoral puncture site, the nurse should check the client's pedal pulses frequently to detect reduced circulation to the feet caused by vascular injury. The nurse also should monitor vital signs for evidence of internal hemorrhage and should observe the puncture site frequently for fresh bleeding. The client should be kept on bed rest for several hours so the puncture site can seal completely. Keeping the client's knee bent is unnecessary. By the time the client returns to the short-procedure unit, manual pressure over the puncture site is no longer needed because a pressure dressing is in place. The nurse shouldn't remove this dressing for several hours — and only if instructed to do so.

33. After undergoing transurethral resection of the prostate to treat benign prostatic hyperplasia, a client returns to the room with continuous bladder irrigation. On the first day after surgery, the client reports bladder pain. What should the nurse do first? 1. Increase the I.V. flow rate. 2. Notify the physician immediately. 3. Assess the irrigation catheter for patency and drainage. 4. Administer morphine sulfate, 2 mg I.V., as prescribed.

Answer 3: RATIONALES: Although postoperative pain is expected, the nurse should make sure that other factors, such as an obstructed irrigation catheter, aren't the cause of the pain. After assessing catheter patency, the nurse should administer an analgesic, such as morphine sulfate, as prescribed. Increasing the I.V. flow rate may worsen the pain. Notifying the physician isn't necessary unless the pain is severe or unrelieved by the prescribed medication.

11. A client with acute pyelonephritis receives a prescription for co-trimoxazole (Septra) P.O. twice daily for 10 days. Which finding best demonstrates that the client has followed the prescribed regimen? 1. Urine output increases to 2,000 ml/day. 2. Flank and abdominal discomfort decrease. 3. Bacteria are absent on urine culture. 4. The red blood cell (RBC) count is normal.

Answer 3: RATIONALES: Co-trimoxazole is a sulfonamide antibiotic used to treat urinary tract infections. Therefore, absence of bacteria on urine culture indicates that the drug has achieved its desired effect. Although flank pain may decrease as the infection resolves, this isn't a reliable indicator of the drug's effectiveness. Co-trimoxazole doesn't affect urine output or the RBC count

21. For a client in the oliguric phase of acute renal failure (ARF), which nursing intervention is most important? 1. Encouraging coughing and deep breathing 2. Promoting carbohydrate intake 3. Limiting fluid intake 4. Providing pain-relief measures

Answer 3: RATIONALES: During the oliguric phase of ARF, urine output decreases markedly, possibly leading to fluid overload. Limiting oral and I.V. fluid intake can prevent fluid overload and its complications, such as heart failure and pulmonary edema. Encouraging coughing and deep breathing is important for clients with various respiratory disorders. Promoting carbohydrate intake may be helpful in ARF but doesn't take precedence over fluid limitation. Controlling pain isn't important because ARF rarely causes pain.

83. A client who has been treated for chronic renal failure (CRF) is ready for discharge. The nurse should reinforce which dietary instruction? 1. "Be sure to eat meat at every meal." 2. "Eat plenty of bananas." 3. "Increase your carbohydrate intake." 4. "Drink plenty of fluids, and use a salt substitute."

Answer 3: RATIONALES: Extra carbohydrates are needed to prevent protein catabolism. In a client with CRF, unrestricted intake of sodium, protein, potassium, and fluid may lead to a dangerous accumulation of electrolytes and protein metabolic products, such as amino acids and ammonia. Therefore, the client must limit intake of sodium; meat, which is high in protein; bananas, which are high in potassium; and fluid, because the failing kidneys can't secrete adequate urine. Salt substitutes are high in potassium and should be avoided.

7. The nurse is caring for a client in acute renal failure. The nurse should expect hypertonic glucose, insulin infusions, and sodium bicarbonate to be used to treat: 1. hypernatremia. 2. hypokalemia. 3. hyperkalemia. 4. hypercalcemia.

Answer 3: RATIONALES: Hyperkalemia is a common complication of acute renal failure. It's life-threatening if immediate action isn't taken to reverse it. The administration of glucose and regular insulin, with sodium bicarbonate if necessary, can temporarily prevent cardiac arrest by moving potassium into the cells and temporarily reducing serum potassium levels. Hypernatremia, hypokalemia, and hypercalcemia don't usually occur with acute renal failure and aren't treated with glucose, insulin, or sodium bicarbonate.

60. Which statement best describes the therapeutic action of loop diuretics? 1. They block reabsorption of potassium on the collecting tubule. 2. They promote sodium secretion into the distal tubule. 3. They block sodium reabsorption in the ascending loop and dilate renal vessels. 4. They promote potassium secretion into the distal tubule and constrict renal vessels.

Answer 3: RATIONALES: Loop diuretics block sodium reabsorption in the ascending loop of Henle, which promotes water diuresis. They also dilate renal vessels. Loop diuretics block potassium reabsorption, but this isn't a therapeutic effect. Thiazide diuretics promote sodium secretion into the distal tubule.

17. The nurse correctly identifies a urine sample with a pH of 4.3 as being which type of solution? 1. Neutral 2. Alkaline 3. Acidic 4. Basic

Answer 3: RATIONALES: Normal urine pH is 4.5 to 8.0; a value of 4.3 reveals acidic urine pH. A pH above 7.0 is considered an alkaline or basic solution. A pH of 7.0 is considered neutral.

101. Which steps should the nurse follow to insert a straight urinary catheter? 1. Create a sterile field, drape the client, clean the meatus, and insert the catheter only 6". 2. Put on gloves, prepare equipment, create a sterile field, expose the urinary meatus, and insert the catheter 6". 3. Prepare the client and equipment, create a sterile field, put on gloves, clean the urinary meatus, and insert the catheter until urine flows. 4. Prepare the client and equipment, create a sterile field, test the catheter balloon, clean the meatus, and insert the catheter until urine flows.

Answer 3: RATIONALES: Option 3 describes all the vital steps for inserting a straight catheter. Option 1 is incorrect because the nurse must prepare the client and equipment before creating a sterile field. Option 2 is incorrect because the nurse put on gloves before creating a sterile field and performing the other tasks. Option 4 describes the procedure for inserting a retention catheter, rather than a straight catheter.

98. Which laboratory value supports a diagnosis of pyelonephritis? 1. Myoglobinuria 2. Ketonuria 3. Pyuria 4. Low white blood cell (WBC) count

Answer 3: RATIONALES: Pyelonephritis is diagnosed by the presence of leukocytosis, hematuria, pyuria, and bacteriuria. The client exhibits fever, chills, and flank pain. Because there is often a septic picture, the WBC count is more likely to be high rather than low, as indicated in option 4. Ketonuria indicates a diabetic state.

82. A client with chronic renal failure (CRF) has developed faulty red blood cell (RBC) production. The nurse should monitor this client for: 1. nausea and vomiting. 2. dyspnea and cyanosis. 3. fatigue and weakness. 4. thrush and circumoral pallor.

Answer 3: RATIONALES: RBCs carry oxygen throughout the body. Decreased RBC production diminishes cellular oxygen, leading to fatigue and weakness. Nausea and vomiting may occur in CRF but don't result from faulty RBC production. Dyspnea and cyanosis are associated with fluid excess, not CRF. Thrush, which signals fungal infection, and circumoral pallor, which reflects decreased oxygenation, aren't signs of CRF.

66. A female client reports to the nurse that she experiences a loss of urine when she jogs. The nurse's assessment reveals no nocturia, burning, discomfort when voiding, or urine leakage before reaching the bathroom. The nurse explains to the client that this type of problem is called: 1. functional incontinence. 2. reflex incontinence. 3. stress incontinence. 4. total incontinence.

Answer 3: RATIONALES: Stress incontinence is a small loss of urine with activities that increase intra-abdominal pressure, such as running, laughing, sneezing, jumping, coughing, and bending. These symptoms occur only in the daytime. Functional incontinence is the inability of a usually continent client to reach the toilet in time to avoid unintentional loss of urine. Reflex incontinence is an involuntary loss of urine at predictable intervals when a specific bladder volume is reached. Total incontinence occurs when a client experiences a continuous and unpredictable loss of urine.

79. To treat a urinary tract infection (UTI), a client is prescribed sulfamethoxazole (Gantanol). The nurse should teach the client that sulfamethoxazole is most likely to cause which adverse effect? 1. Anxiety 2. Headache 3. Diarrhea 4. Dizziness

Answer 3: RATIONALES: Sulfamethoxazole is most likely to cause diarrhea. Nausea and vomiting are other common adverse effects. This drug rarely causes anxiety, headache, or dizziness.

59. The registered nurse and nursing assistant are caring for a group of clients. Which client's care can safely be delegated to the nursing assistant? 1. A 35-year-old client who underwent surgery 12 hours ago and has a suprapubic catheter in place that is draining burgundy colored urine 2. A 63-year-old client with uncontrolled diabetes mellitus who underwent radical suprapubic prostatectomy 1 day ago and has an indwelling urinary catheter draining yellow urine with clots 3. A 45-year-old client diagnosed with renal calculi who must ambulate four times daily and drink plenty of fluids. 4. A 19-year-old client who requires neurological assessment every four hours after sustaining a spinal cord injury in a motor vehicle accident that left him with paraplegia

Answer 3: RATIONALES: The care of the client in option 3 can safely be delegated to the nursing assistant. The client in option 1 had surgery 12 hours ago; therefore, the registered nurse should care for the client because the client requires close assessment. The client in option 2 also requires careful assessment by the registered nurse because the client's diabetes mellitus is uncontrolled. In addition, the registered nurse should care for the client in option 4 because the client requires neurological assessment, which isn't within the scope of practice for the nursing assistant.

13. The nurse is caring for a client diagnosed with acute renal failure. The nurse notes on the intake and output record that the total urine output for the previous 24 hours was 95 ml. Urine output that's less than 100 ml in 24 hours is known as: 1. oliguria. 2. polyuria. 3. anuria. 4. hematuria.

Answer 3: RATIONALES: Urine output less than 100 ml in 24 hours is called anuria. Urine output of less than 400 ml but more than 100 ml in 24 hours is called oliguria. Polyuria is excessive urination. Hematuria is the presence of blood in the urine.

64. The nurse is providing postprocedure care for a client who underwent percutaneous lithotripsy. In this procedure, an ultrasonic probe inserted through a nephrostomy tube into the renal pelvis generates ultra-high-frequency sound waves to shatter renal calculi. The nurse should instruct the client to: 1. limit oral fluid intake for 1 to 2 weeks. 2. report the presence of fine, sandlike particles through the nephrostomy tube. 3. notify the physician about cloudy or foul-smelling urine. 4. report bright pink urine within 24 hours after the procedure.

Answer 3: RATIONALES: The client should report the presence of foul-smelling or cloudy urine. Unless contraindicated, the client should be instructed to drink large quantities of fluid each day to flush the kidneys. Sandlike debris is normal because of residual stone products. Hematuria is common after lithotripsy.

72. After undergoing retropubic prostatectomy, a client returns to his room. The client is on nothing-by-mouth status and has an I.V. infusing in his right forearm at a rate of 100 ml/hour. The client also has an indwelling urinary catheter that is draining light pink urine. While assessing the client, the nurse notes that his urine output is red and has dropped to 15 ml and 10 ml for the last two consecutive hours. How can the nurse best explain this drop in urine output? 1. It's a normal finding caused by blood loss during surgery. 2. It's a normal finding associated with the client's nothing-by-mouth status. 3. It's an abnormal finding that requires further assessment. 4. It's an abnormal finding that will correct itself when the client ambulates.

Answer 3: RATIONALES: The drop in urine output to less than 30 ml/hour is abnormal and requires further assessment. The reduction in urine output may be caused by an obstruction in the urinary catheter tubing or deficient fluid volume from blood loss. The client's nothing-by-mouth status isn't the cause of the low urine output because the client is receiving I.V. fluid to compensate for the lack of oral intake. Ambulation promotes urination; however, the client should produce at least 30 ml of urine/hour

70. A client with suspected renal insufficiency is scheduled for a comprehensive diagnostic workup. After the nurse explains the diagnostic tests, the client asks which part of the kidney "does the work." Which answer is correct? 1. The glomerulus 2. Bowman's capsule 3. The nephron 4. The tubular system

Answer 3: RATIONALES: The nephron is the functioning unit of the kidney. The glomerulus, Bowman's capsule, and tubular system are components of the nephron.

61. A client with chronic renal failure (CRF) is admitted to the urology unit. Which diagnostic test results are consistent with CRF? 1. Increased pH with decreased hydrogen ions 2. Increased serum levels of potassium, magnesium, and calcium 3. Blood urea nitrogen (BUN) 100 mg/dl and serum creatinine 6.5 mg/dl 4. Uric acid analysis 3.5 mg/dl and phenolsulfonphthalein (PSP) excretion 75%

Answer 3: RATIONALES: The normal BUN level ranges 8 to 23 mg/dl; the normal serum creatinine level ranges from 0.7 to 1.5 mg/dl. The test results in option 3 are abnormally elevated, reflecting CRF and the kidneys' decreased ability to remove nonprotein nitrogen waste from the blood. CRF causes decreased pH and increased hydrogen ions — not vice versa. CRF also increases serum levels of potassium, magnesium, and phosphorous, and decreases serum levels of calcium. A uric acid analysis of 3.5 mg/dl falls within the normal range of 2.7 to 7.7 mg/dl; PSP excretion of 75% also falls within the normal range of 60% to 75%.

37. A client is scheduled for a renal arteriogram. When the nurse checks the chart for allergies to shellfish or iodine, the nurse finds no allergies recorded. The client is unable to provide the information. During the procedure, the nurse should be alert for which finding that may indicate an allergic reaction to the dye used during the arteriogram. 1. Increased alertness 2. Hypoventilation 3. Pruritus 4. Unusually smooth skin

Answer 3: RATIONALES: The nurse should be alert for urticaria and pruritus, which may indicate a mild anaphylactic reaction to the arteriogram dye. Decreased (not increased) alertness may occur as well as dyspnea (not hypoventilation). Unusually smooth skin isn't a sign of anaphylaxis.

92. A 75-year-old client undergoes total hip replacement. After surgery, the client questions why she must go to a rehabilitation center because she has family who can care for her. Which response by the nurse is best? 1. You'll need help with your bath and meals for quite some time." 2. "The rehabilitation staff can provide you with better care." 3. "The rehabilitation staff can evaluate your progress and make sure that you exercise without risking injury." 4. "The doctor wants you to go to the rehabilitation center until you're fully recovered and able to care for yourself."

Answer 3: RATIONALES: The nurse should respond by emphasizing that the rehabilitation center can evaluate progress and make sure that exercises are performed without risking injury. This response points out that the goal of rehabilitation is safely achieving mobility, not providing total care. Option 1 doesn't provide adequate information about the role of rehabilitation or the client's future needs. The rehabilitation center will help the client learn to bathe herself. Option 2 is judgmental about care the family might provide and doesn't adequately explain the role of a rehabilitation center. Option 4 doesn't explain the importance of a rehabilitation center.

62. A client with renal failure is undergoing continuous ambulatory peritoneal dialysis. Which nursing diagnosis is most appropriate for this client? 1. Impaired urinary elimination 2. Toileting self-care deficit 3. Risk for infection 4. Activity intolerance

Answer 3: RATIONALES: The peritoneal dialysis catheter and regular exchanges of the dialysis bag provide a direct portal for bacteria to enter the body. Therefore, the client is at risk for infection. If the client experiences repeated peritoneal infections, continuous ambulatory peritoneal dialysis may no longer be effective in clearing waste products. The other options may be pertinent but are secondary to the risk for infection.

94. The nurse is planning a group teaching session on the topic of urinary tract infection (UTI) prevention. Which point would the nurse want to include? 1. Limit fluid intake to reduce the need to urinate. 2. Take medication prescribed for a UTI until the symptoms subside. 3. Notify the physician if urinary urgency, burning, frequency, or difficulty occurs. 4. Wear only nylon underwear to reduce the chance of irritation.

Answer 3: RATIONALES: Urgency, burning, frequency, and difficulty urinating are all common symptoms of a UTI. The client should notify his physician so that a microscopic urinalysis can be done and appropriate treatment can be initiated. The client should be instructed to drink 2 to 3 L of fluid per day to dilute the urine and reduce irritation of the bladder mucosa. The full amount of antibiotics prescribed for UTIs must be taken despite the fact that the symptoms may have subsided. This will help to prevent recurrences of UTI. Women are told to avoid scented toilet tissue and bubble baths and to wear cotton underwear, not nylon, to reduce the chance of irritation.

35. A client with chronic renal failure has a serum potassium level of 6.8 mEq/L. What should the nurse assess first? 1. Blood pressure 2. Respirations 3. Temperature 4. Pulse

Answer 4 RATIONALES: An elevated serum potassium level may lead to a life-threatening cardiac arrhythmia, which the nurse can detect immediately by palpating the pulse. The client's blood pressure may change, but only as a result of the arrhythmia. Therefore, the nurse should assess blood pressure later. The nurse also can delay assessing respirations and temperature because these aren't affected by the serum potassium level.

36. The nurse is assessing a male client diagnosed with gonorrhea. Which symptom most likely prompted the client to seek medical attention? 1. Rashes on the palms of the hands and soles of the feet 2. Cauliflower-like warts on the penis 3. Painful red papules on the shaft of the penis 4. Foul-smelling discharge from the penis

Answer 4 RATIONALES: Symptoms of gonorrhea in men include purulent, foul-smelling drainage from the penis and painful urination. Rashes on the palms of the hands and soles of the feet are symptoms of the secondary stage of syphilis. Cauliflower-like warts on the penis are a sign of human papillomavirus. Painful red papules on the shaft of the penis may be a sign of the first stage of genital herpes.

42. A client with a history of chronic cystitis comes to the outpatient clinic with signs and symptoms of this disorder. To prevent cystitis from recurring, the nurse recommends maintaining an acid-ash diet to acidify the urine, thereby decreasing the rate of bacterial multiplication. On an acid-ash diet, the client must restrict which beverage? 1. Cranberry juice 2. Coffee 3. Prune juice 4. Milk

Answer 4: RATIONALES: A client on an acid-ash diet must avoid milk and milk products because these make the urine more alkaline, encouraging bacterial growth. Other foods to avoid on this diet include all vegetables except corn and lentils; all fruits except cranberries, plums, and prunes; and any food containing large amounts of potassium, sodium, calcium, or magnesium. Cranberry and prune juice are encouraged because they acidify the urine. Coffee and tea are considered neutral because they don't alter the urine pH.

81. When performing a scrotal examination, the nurse finds a nodule. What should the nurse do next? 1. Notify the physician. 2. Change the client's position and repeat the examination. 3. Perform a rectal examination. 4. Transilluminate the scrotum.

Answer 4: RATIONALES: A nurse who discovers a nodule, swelling, or other abnormal finding during a scrotal examination should transilluminate the scrotum by darkening the room and shining a flashlight through the scrotum behind the mass. A scrotum filled with serous fluid transilluminates as a red glow; a more solid lesion, such as a hematoma or mass, doesn't transilluminate and may appear as a dark shadow. Although the nurse should notify the physician of the abnormal finding, performing transillumination first provides additional information. The nurse can't uncover more information about a scrotal mass by changing the client's position and repeating the examination or by performing a rectal examination.

12. A client admitted with a gunshot wound to the abdomen is transferred to the intensive care unit after an exploratory laparotomy. I.V. fluid is being infused at 150 ml/hr. Which assessment finding suggests that the client is experiencing acute renal failure (ARF)? 1. Blood urea nitrogen (BUN) level of 22 mg/dl 2. Serum creatinine level of 1.2 mg/dl 3. Temperature of 100.2° F (37.8° C) 4. Urine output of 250 ml/24 hours

Answer 4: RATIONALES: ARF, characterized by abrupt loss of kidney function, commonly causes oliguria, which is demonstrated by a urine output of 250 ml/24 hours. A serum creatinine level of 1.2 mg/dl isn't diagnostic of ARF. A BUN level of 22 mg/dl or a temperature of 100.2° F (37.8° C) wouldn't result from this disorder.

30. During a routine examination, the nurse notes that the client seems unusually anxious. Anxiety can affect the genitourinary system by: 1. slowing the glomerular filtration rate. 2. increasing sodium resorption. 3. decreasing potassium excretion. 4. stimulating or hindering micturition.

Answer 4: RATIONALES: Anxiety may stimulate or hinder micturition. Its most noticeable effect is to cause frequent voiding and urinary urgency. However, when anxiety leads to generalized muscle tension, it may hinder urination because the perineal muscles must relax to complete micturition. Anxiety doesn't slow the glomerular filtration rate, increase sodium resorption, or decrease potassium excretion.

39. A client requires hemodialysis. Which type of drug should be withheld before this procedure? 1. Phosphate binders 2. Insulin 3. Antibiotics 4. Cardiac glycosides

Answer 4: RATIONALES: Cardiac glycosides such as digoxin should be withheld before hemodialysis. Hypokalemia is one of the electrolyte shifts that occur during dialysis, and a hypokalemic client is at risk for arrhythmias secondary to digitalis toxicity. Phosphate binders and insulin can be administered because they aren't removed from the blood by dialysis. Some antibiotics are removed by dialysis and should be administered after the procedure to ensure their therapeutic effects. The nurse should check a formulary to determine whether a particular antibiotic should be administered before or after dialysis.

73. The nurse is teaching a client with genital herpes. Education for this client should include an explanation of: 1. the need for the use of petroleum products. 2. why the disease is transmittable only when visible lesions are present. 3. the option of disregarding safer-sex practices now that he's already infected. 4. the importance of informing his partners of the disease.

Answer 4: RATIONALES: Clients with genital herpes should inform their partners of the disease. Petroleum products should be avoided because they can cause the virus to spread. The notion that genital herpes is only transmittable when visible lesions are present is false. Anyone not in a long-term, monogamous relationship, regardless of current health status, should follow safer-sex practices

71. The nurse is caring for a client with acute renal failure. The nurse should expect that hypertonic glucose, insulin infusions, and sodium bicarbonate will be used to treat what complication of acute renal failure? 1. Hypokalemia 2. Hyperphosphatemia 3. Hypophosphatemia 4. Hyperkalemia

Answer 4: RATIONALES: Hyperkalemia is a common complication of acute renal failure. The administration of glucose and regular insulin infusions, with sodium bicarbonate if necessary, can temporarily prevent cardiac arrest by moving potassium into the cells and temporarily reducing potassium levels. This treatment isn't used to treat hyperphosphatemia or hypophosphatemia.

54. The client underwent a transurethral resection of the prostate gland 24 hours ago and is on continuous bladder irrigation. Which nursing intervention is appropriate? 1. Tell the client to try to urinate around the catheter to remove blood clots. 2. Restrict fluids to prevent the client's bladder from becoming distended. 3. Prepare to remove the catheter. 4. Use aseptic technique when irrigating the catheter.

Answer 4: RATIONALES: If the catheter is blocked by blood clots, it may be irrigated according to physician's orders or facility protocol. The nurse should use sterile technique to reduce the risk of infection. Urinating around the catheter can cause painful bladder spasms. Encourage the client to drink fluids to dilute the urine and maintain urine output. The catheter remains in place for 2 to 4 days after surgery and is only removed with a physician's order.

4. A client with decreased urine output refractory to fluid challenges is evaluated for renal failure. Which condition may cause the intrinsic (intrarenal) form of acute renal failure? 1. Poor perfusion to the kidneys 2. Damage to cells in the adrenal cortex 3. Obstruction of the urinary collecting system 4. Nephrotoxic injury secondary to use of contrast media

Answer 4: RATIONALES: Intrinsic renal failure results from damage to the kidney, such as from nephrotoxic injury caused by contrast media, antibiotics, corticosteroids, or bacterial toxins. Poor perfusion to the kidneys may result in prerenal failure. Damage to the epithelial cells of the renal tubules results from nephrotoxic injury, not damage to the adrenal cortex. Obstruction of the urinary collecting system may cause postrenal failure.

55. A 25-year-old female client seeks care for a possible infection. Her symptoms include burning on urination and frequent, urgent voiding of small amounts of urine. She's placed on trimethoprim-sulfamethoxazole (Bactrim) to treat possible infection. Another medication is prescribed to decrease the pain and frequency. Which is the most likely medication prescribed for the pain? 1. nitrofurantoin (Macrodantin) 2. ibuprofen (Motrin) 3. acetaminophen with codeine 4. phenazopyridine (Pyridium)

Answer 4: RATIONALES: Phenazopyridine may be prescribed in conjunction with an antibiotic for painful bladder infections to promote comfort. Because of its local anesthetic action on the urinary mucosa, phenazopyridine specifically relieves bladder pain. Nitrofurantoin is a urinary antiseptic with no analgesic properties. Although ibuprofen and acetaminophen with codeine are analgesics, they don't exert a direct effect on the urinary mucosa.

27. A client with heart failure admitted to an acute care facility and is found to have a cystocele. When planning care for this client, the nurse is most likely to formulate which nursing diagnosis? 1. Total urinary incontinence 2. Functional urinary incontinence 3. Reflex urinary incontinence 4. Stress urinary incontinence

Answer 4: RATIONALES: Stress urinary incontinence is a urinary problem associated with cystocele — herniation of the bladder into the birth canal. Other problems associated with this disorder include urinary frequency, urinary urgency, urinary tract infection (UTI), and difficulty emptying the bladder. Total incontinence, functional incontinence, and reflex incontinence usually result from neurovascular dysfunction, not cystocele.

6. A client is admitted with nausea, vomiting, and diarrhea. His blood pressure on admission is 74/30 mm Hg. The client is oliguric and his blood urea nitrogen (BUN) and creatinine levels are elevated. The physician will most likely write an order for which treatment? 1. Force oral fluids. 2. Administer furosemide (Lasix) 20 mg I.V. 3. Start hemodialysis after a temporary access is obtained. 4. Start I.V. fluids with a normal saline solution bolus followed by a maintenance dose.

Answer 4: RATIONALES: The client is in prerenal failure caused by hypovolemia. I.V. fluids should be given with a bolus of normal saline solution followed by maintenance I.V. therapy. This treatment should rehydrate the client, causing his blood pressure to rise, his urine output to increase, and the BUN and creatinine levels to normalize. The client wouldn't be able to tolerate oral fluids because of the nausea, vomiting, and diarrhea. The client isn't fluid-overloaded so his urine output won't increase with furosemide, which would actually worsen the client's condition. The client doesn't require dialysis because the oliguria and elevated BUN and creatinine levels are caused by dehydration.

89. A client is frustrated and embarrassed by urinary incontinence. Which measure should the nurse include in a bladder retraining program? 1. Establishing a predetermined fluid intake pattern for the client 2. Encouraging the client to increase the time between voidings 3. Restricting fluid intake to reduce the need to void 4. Assessing present elimination patterns

Answer 4: RATIONALES: The guidelines for initiating bladder retraining include assessing the client's intake patterns, voiding patterns, and reasons for each accidental voiding. Lowering the client's fluid intake won't reduce or prevent incontinence. The client should actually be encouraged to drink 1.5 to 2 L of water per day. A voiding schedule should be established after assessment.

40. The physician prescribes a single dose of trimethoprim/sulfamethoxazole (Bactrim) by mouth for a client diagnosed with an uncomplicated urinary tract infection (UTI). The pharmacy sends three unit-dose tablets. The nurse verifies the physician's order. What should the nurse do next? 1. Administer the three tablets as the single dose. 2. Call the physician to verify the order. 3. Give one tablet, three times per day. 4. Call the hospital pharmacist and question the medication supplied.

Answer 4: RATIONALES: The nurse should call the hospital pharmacy and question the medication supplied. The hospital pharmacist should be able to tell the nurse whether three tablets are necessary for the single dose or whether a dispensing error occurred. It isn't clear whether the three tablets are the single dose because they were packaged as a unit-dose. The physician order was clearly written, so clarifying the order with the physician isn't necessary. Administering the tablets without clarification might cause a medication error.

45. The client is scheduled for urinary diversion surgery to treat bladder cancer. Before surgery, the health care team consisting of a nurse, dietician, social worker, enterostomal therapist, surgeon, client educator, and mental health worker meet with the client. After the meeting, the client states, "My life won't ever be the same. What am I going to do?" Which health team member should the nurse consult to help with the client's concerns? 1. Social worker 2. Surgeon 3. Dietician 4. Client educator

Answer 4: RATIONALES: The nurse should consult the client educator to help the client with his fears and concerns. Providing the client with information can greatly allay the client's fears. The social worker can provide the client with services he may need after discharge. The dietician can help with dietary concerns but can't provide help with direct concerns about the surgery.

78. The nurse is providing inservice education for the staff about evidence collection after sexual assault. The educational session is successful when the staff focuses their initial care on which step? 1. Collecting semen 2. Performing the pelvic examination 3. Obtaining consent for examination 4. Supporting the client's emotional status

Answer 4: RATIONALES: The teaching session is successful when the nurses focus on supporting the client's emotional status first. Next, the nurses should gain consent to perform the pelvic examination, perform the examination, and collect evidence, such as semen if present.

22. A client is admitted for treatment of chronic renal failure (CRF). The nurse knows that this disorder increases the client's risk of: 1. water and sodium retention secondary to a severe decrease in the glomerular filtration rate. 2. a decreased serum phosphate level secondary to kidney failure. 3. an increased serum calcium level secondary to kidney failure. 4. metabolic alkalosis secondary to retention of hydrogen ions.

Answer: 1 RATIONALES: A client with CRF is at risk for fluid imbalance — dehydration if the kidneys fail to concentrate urine, or fluid retention if the kidneys fail to produce urine. Electrolyte imbalances associated with this disorder result from the kidneys' inability to excrete phosphorus; such imbalances may lead to hyperphosphatemia with reciprocal hypocalcemia. CRF may cause metabolic acidosis, not metabolic alkalosis, secondary to inability of the kidneys to excrete hydrogen ions.

24. A female client has just been diagnosed with condylomata acuminata (genital warts). What information is appropriate to tell this client? 1. This condition puts her at a higher risk for cervical cancer; therefore, she should have a Papanicolaou (Pap) smear annually. 2. The most common treatment is metronidazole (Flagyl), which should eradicate the problem within 7 to 10 days. 3. The potential for transmission to her sexual partner will be eliminated if condoms are used every time she and her partner have sexual intercourse. 4. The human papillomavirus (HPV), which causes condylomata acuminata, can't be transmitted during oral sex.

Answer: 1 RATIONALES: Women with condylomata acuminata are at risk for cancer of the cervix and vulva. Yearly Pap smears are very important for early detection. Because condylomata acuminata is a virus, there is no permanent cure. Because condylomata acuminata can occur on the vulva, a condom won't protect sexual partners. HPV can be transmitted to other parts of the body, such as the mouth, oropharynx, and larynx.

2. After a retropubic prostatectomy, a client needs continuous bladder irrigation. The client has an I.V. of D5W infusing at 40 ml/hr, and a triple-lumen urinary catheter with normal saline solution infusing at 200 ml/hr. The nurse empties the urinary catheter drainage bag three times during an 8-hour period for a total of 2,780 ml. How many milliliters does the nurse calculate as urine?

Answer: 1180 RATIONALES: During 8 hours, 1,600 ml of bladder irrigation has been infused (200 ml × 8 hr = 1,600 ml/8 hr). The nurse then subtracts this amount of infused bladder irrigation from the total volume in the drainage bag (2,780 ml − 1,600 ml = 1,180 ml) to determine urinary output.

49. Which laboratory test is the most accurate indicator of a client's renal function? 1. Blood urea nitrogen 2. Creatinine clearance 3. Serum creatinine 4. Urinalysis

Answer: 2 RATIONALES: Creatinine clearance is the most accurate indicator of a client's renal function because it closely correlates with the kidney's glomerular filtration rate and tubular excretion ability. Results from the other options may be influenced by various conditions and aren't specific to renal disease.

3. The nurse is caring for a client with acute pyelonephritis. Which nursing intervention is most important? 1. Administering a sitz bath twice per day 2. Increasing fluid intake to 3 L/day 3. Using an indwelling urinary catheter to measure urine output accurately 4. Encouraging the client to drink cranberry juice to acidify the urine

Answer: 2 Answer 2: RATIONALES: Acute pyelonephritis is a sudden inflammation of the interstitial tissue and renal pelvis of one or both kidneys. Infecting bacteria are normal intestinal and fecal flora that grow readily in urine. Pyelonephritis may result from procedures that involve the use of instruments (such as catheterization, cystoscopy, and urologic surgery) or from hematogenic infection. The most important nursing intervention is to increase fluid intake to 3 L/day. This helps empty the bladder of contaminated urine and prevents calculus formation. Administering a sitz bath would increase the likelihood of fecal contamination. Using an indwelling urinary catheter could cause further contamination. Encouraging the client to drink cranberry juice to acidify urine is helpful but isn't the most important interaction.

23. During rounds, a client admitted with gross hematuria asks the nurse about the physician's diagnosis. To facilitate effective communication, what should the nurse do? 1. Ask why the client is concerned about the diagnosis. 2. Change the subject to something more pleasant. 3. Provide privacy for the conversation. 4. Give the client some good advice.

Answer: 3 RATIONALES: Providing privacy for the conversation is a form of active listening, which focuses solely on the client's needs. Asking why the client is concerned, changing the subject, or giving advice tends to block therapeutic communication.

52. A client develops decreased renal function and requires a change in antibiotic dosage. On which factor would the physician base the dosage change? 1. GI absorption rate 2. Therapeutic index 3. Creatinine clearance 4. Liver function studies

Answer: 3 RATIONALES: The physician orders tests for creatinine clearance to gauge the kidney's glomerular filtration rate; this is important because most drugs are excreted at least partially by the kidneys. The GI absorption rate, therapeutic index, and liver function studies don't help determine dosage change in a client with decreased renal function

14. A patient with a history of renal calculi is hospitalized with gross hematuria and severe colicky left flank pain that radiates to his left testicle. In planning care for the patient, the nurse gives the highest priority to the nursing diagnosis of a. acute pain related to irritation by the stone. b. deficient fluid volume related to inadequate intake. c. risk for infection related to urinary system damage. d. risk for nausea related to pain and renal colic.

Answer: A Rationale: Although all the diagnoses are appropriate, the initial nursing actions should focus on relief of the acute pain. Cognitive Level: Application Text Reference: p. 1173 Nursing Process: Diagnosis NCLEX: Physiological Integrity

2. Trimethoprim and sulfamethoxazole (Bactrim) BID for 7 days is ordered for a patient who has a recurrent relapse of an Escherichia coli UTI. The nurse instructs the patient to a. take the antibiotic for the full 7 days, even if symptoms improve in a few days. b. return to the clinic in 3 days so that a urine culture can be done to evaluate the effectiveness of the drug. c. increase the effectiveness of the drug by taking it with cranberry juice to acidify the urine. d. take two of the pills a day for 5 days, and reserve the rest of the pills to take if the symptoms reappear.

Answer: A Rationale: Although an initial infection may be treated with a shorter course of antibiotics, the patient with a recurrent infection should take the antibiotic for 7 days. Success of treatment is evaluated by resolution of symptoms rather than by a repeat culture. Acidifying the urine when a patient is taking sulfa antibiotics may lead to stone formation. The patient is instructed to take all the antibiotics. Cognitive Level: Application Text Reference: p. 1157 Nursing Process: Implementation NCLEX: Physiological Integrity

26. Following rectal surgery, a patient voids about 50 ml of urine every 30 to 60 minutes. Which nursing action is most appropriate? a. Use an ultrasound scanner to check for residual urine after voiding. b. Have the patient take small amounts of fluid frequently throughout the day. c. Reassure the patient that this is normal after rectal surgery due to anesthesia. d. Monitor the patient's intake and output over the next few hours.

Answer: A Rationale: An ultrasound scanner can be used to check for residual urine after the patient voids. Because the patient's history and clinical manifestations are consistent with overflow incontinence, it is not appropriate to have the patient drink small amounts. Although overflow incontinence is not unusual after surgery, the nurse should intervene to correct the physiologic problem, not just reassure the patient. The patient may develop reflux into the renal pelvis as well as discomfort from a full bladder if the nurse waits to address the problem for several hours. Cognitive Level: Application Text Reference: p. 1182 Nursing Process: Implementation NCLEX: Physiological Integrity

16. A patient with a confirmed urinary stone in the proximal left ureter undergoes extracorporeal shock-wave lithotripsy. Which information is most important for the nurse to collect after lithotripsy? a. Urine output b. Pain level c. Appearance of the site d. Patient temperature

Answer: A Rationale: Because lithotripsy breaks the stone into small sand, which could cause obstruction, it is important to monitor the urine output. The patient may have pain as the stones pass and bruising at the site, but these are not unexpected. Extracorporeal shock wave lithotripsy (ESWL) is not associated with a risk for infection. Cognitive Level: Application Text Reference: p. 1172 Nursing Process: Assessment NCLEX: Physiological Integrity

27. A patient in the hospital has a history of urinary incontinence. Which nursing action will be included in the plan of care? a. Place a bedside commode near the patient's bed. b. Use an ultrasound scanner to check urine residual after the patient voids. c. Demonstrate the use of the Credé maneuver to the patient. d. Teach the use of Kegel exercises to strengthen the pelvic floor.

Answer: A Rationale: Environmental changes can make it easier for the patient to avoid incontinence for patients with urinary incontinence. Checking for residual urine and performing the Credé maneuver are interventions for overflow incontinence. Kegel exercises are useful for stress incontinence. Cognitive Level: Application Text Reference: p. 1181 Nursing Process: Planning NCLEX: Physiological Integrity

25. After her bath, a 62-year-old patient asks the nurse for a perineal pad, saying that she uses them because sometimes she leaks urine when she laughs or coughs. Which intervention is most appropriate to include in the care plan for the patient? a. Teach the patient how to perform Kegel exercises. b. Demonstrate how to perform Credé's maneuver. c. Place commode at the patient's bedside. d. Assist the patient to the bathroom q3hr.

Answer: A Rationale: Exercises to strengthen the pelvic floor muscles will help reduce stress incontinence. The Credé maneuver is used to help empty the bladder for patients with overflow incontinence. Placing the commode close to the bedside and assisting the patient to the bathroom are helpful for functional incontinence. Cognitive Level: Application Text Reference: pp. 1181-1184 Nursing Process: Planning NCLEX: Health Promotion and Maintenance

23. Following an open-loop resection and fulguration of the bladder, a patient is unable to void. Which nursing action should be implemented? a. Assist the patient to take a 15-minute sitz bath. b. Encourage the patient to drink several glasses of water. c. Teach the patient how to do isometric perineal exercises. d. Insert a straight catheter and drain the bladder.

Answer: A Rationale: Sitz baths will relax the perineal muscles and promote voiding. Although the patient should be encouraged to drink fluids, this would not be appropriate when the patient is experiencing retention. Kegel exercises are helpful in the prevention of incontinence. Catheter insertion increases the risk for infection. Cognitive Level: Application Text Reference: p. 1179 Nursing Process: Implementation NCLEX: Physiological Integrity

37. A 26-year-old patient with a history of polycystic kidney disease is admitted to the surgical unit after having knee surgery. Which of the routine postoperative orders is most important for the nurse discuss with the health care provider? a. Give ketorolac (Toradol) 10 mg PO PRN for pain. b. Infuse 5% dextrose in normal saline at 75 ml/hr. c. Obtain BUN, creatinine, and electrolytes in 2 hours. d. Order regular diet after patient is awake and alert.

Answer: A Rationale: The NSAIDs should be avoided in patients with decreased renal function because nephrotoxicity is a potential adverse effect. The other orders do not need any clarification or change. Cognitive Level: Application Text Reference: p. 1173 Nursing Process: Implementation NCLEX: Physiological Integrity

11. A patient who is diagnosed with nephrotic syndrome has 3+ ankle and leg edema and ascites. Which nursing diagnosis is a priority for the patient? a. Fluid-volume excess related to low serum protein levels b. Altered nutrition: less than required related to protein restriction c. Activity intolerance related to increased weight and fatigue d. Disturbed body image related to peripheral edema and ascites

Answer: A Rationale: The patient has massive edema, so the priority problem at this time is the excess of fluid volume. The other nursing diagnoses are also appropriate, but the focus of nursing care should be resolution of the edema and ascites. Cognitive Level: Application Text Reference: pp. 1167-1168 Nursing Process: Diagnosis NCLEX: Physiological Integrity

36. A patient with bladder cancer is scheduled for intravesical chemotherapy. In preparation for the treatment the nurse will teach the patient about a. the need to empty the bladder prior to treatment. b. premedicating to prevent nausea. c. the importance of oral care during treatment. d. where to obtain wigs and scarves.

Answer: A Rationale: The patient will be asked to empty the bladder before instillation of the chemotherapy. Systemic side effects are not experienced with intravesical chemotherapy. Cognitive Level: Application Text Reference: p. 1180 Nursing Process: Implementation NCLEX: Physiological Integrity

3. The nurse determines that instruction regarding prevention of future UTIs for a patient with cystitis has been effective when the patient states, a. "I will empty my bladder every 3 to 4 hours during the day." b. "I can use vaginal sprays to reduce bacteria." c. "I will wash with soap and water before sexual intercourse." d. "I will drink a quart of water or other fluids every day."

Answer: A Rationale: Voiding every 3 to 4 hours is recommended to prevent UTIs. Use of vaginal sprays is discouraged. The bladder should be emptied before and after intercourse, but cleaning with soap and water is not necessary. A quart of fluids is insufficient to provide adequate urine output to decrease risk for UTI. Cognitive Level: Application Text Reference: p. 1161 Nursing Process: Evaluation NCLEX: Health Promotion and Maintenance

30. A patient undergoes a nephrectomy for massive trauma to the kidney resulting from a fall from a scaffold. Which assessment data obtained postoperatively are most important to communicate to the surgeon? a. Blood pressure is 102/48. b. Urine output is 20 ml/hr for 2 hours. c. Crackles are heard at both lung bases. d. Incisional pain level is 8/10.

Answer: B Rationale: Because the urine output should be at least 0.5 ml/kg/hr, a 40-ml output for 2 hours indicates that the patient may have decreased renal perfusion because of bleeding, inadequate fluid intake, or obstruction at the suture site. The blood pressure requires ongoing monitoring but does not indicate inadequate perfusion at this time. The patient should cough and deep breathe, but the crackles do not indicate a need for an immediate change in therapy. The incisional pain should be addressed, but this is not as potentially life-threatening as decreased renal perfusion. In addition, the nurse can medicate the patient for pain. Cognitive Level: Application Text Reference: p. 1188 Nursing Process: Assessment NCLEX: Physiological Integrity

7. After teaching a patient with interstitial cystitis about management of the condition, the nurse determines that further instruction is needed when the patient says, a. "I will have to stop having coffee and orange juice for breakfast." b. "I should start taking a high-potency multiple vitamin every morning." c. "I should call the doctor about increased bladder pain or odorous urine." d. "I will buy some calcium glycerophosphate (Prelief) at the pharmacy."

Answer: B Rationale: High-potency multiple vitamins may irritate the bladder and increase symptoms. The other patient statements indicate good understanding of the teaching. Cognitive Level: Application Text Reference: p. 1164 Nursing Process: Evaluation NCLEX: Physiological Integrity

5. A 34-year-old patient with diabetes mellitus is hospitalized with fever, anorexia, and confusion. The health care provider suspects acute pyelonephritis when the urinalysis reveals bacteriuria. An appropriate collaborative problem identified by the nurse for the patient is potential complication a. hydronephrosis. b. urosepsis. c. acute renal failure. d. chronic pyelonephritis.

Answer: B Rationale: Infection can easily spread from the kidney to the circulation, causing urosepsis. A patient with a urinary tract obstruction will be at risk for hydronephrosis. Acute renal failure is not a common complication of acute pyelonephritis unless urosepsis and septic shock develop. Chronic pyelonephritis may occur after recurrent upper UTIs. Cognitive Level: Application Text Reference: p. 1161 Nursing Process: Diagnosis NCLEX: Physiological Integrity

17. The composition of a patient's renal calculus is identified as uric acid. To prevent recurrence of stones, the nurse teaches the patient to avoid a. spinach, chocolate, and tomatoes. b. organ meats and fish with fine bones. c. milk and dairy products. d. legumes and dried fruits.

Answer: B Rationale: Organ meats and fish such as sardines increase purine levels and uric acid. Spinach, chocolate, and tomatoes should be avoided in patients who have oxalate stones. Milk, dairy products, legumes, and dried fruits may increase the incidence of calcium-containing stones. Cognitive Level: Application Text Reference: pp. 1170-1171 Nursing Process: Implementation NCLEX: Health Promotion and Maintenance

1. When assessing the patient who has a lower urinary tract infection (UTI), the nurse will initially ask about a. flank pain. b. pain with urination. c. poor urine output. d. nausea.

Answer: B Rationale: Pain with urination is a common symptom of a lower UTI. Urine output does not decrease, but frequency may be experienced. Flank pain and nausea are associated with an upper UTI. Cognitive Level: Application Text Reference: p. 1157 Nursing Process: Assessment NCLEX: Physiological Integrity

32. A patient has a cystectomy and a Kock continent diversion created for treatment of bladder cancer. During postoperative teaching of the patient, it is important that the nurse include instructions regarding a. application of ostomy appliances. b. catheterization technique and schedule. c. use of barrier products for skin protection. d. analgesic use before emptying the pouch.

Answer: B Rationale: The Kock pouch enables the patient to self-catheterize every 4 to 6 hours. There is no need for an ostomy device or barrier products. Catheterization of the pouch is not painful. Cognitive Level: Application Text Reference: p. 1190 Nursing Process: Implementation NCLEX: Physiological Integrity

29. The nurse observes a nursing assistant (NA) doing all of the following when caring for a patient with a retention catheter. Which action requires that the nurse intervene? a. The NA uses an alcohol-based hand cleaner before performing catheter care. b. The NA disconnects the catheter from the drainage tube to obtain a specimen. c. The NA uses soap and water when cleaning around the urinary meatus. d. The NA tapes the catheter to the skin on the patient's upper inner thigh.

Answer: B Rationale: The catheter should not be disconnected from the drainage tube because this increases the risk for UTI. The other actions are appropriate and do not require any intervention. Cognitive Level: Application Text Reference: p. 1186 Nursing Process: Assessment NCLEX: Physiological Integrity

15. The nurse instructs a patient seen in the outpatient clinic with symptoms of renal calculi to strain all urine and to a. report the pain level when the stone passed. b. collect the stone and bring it to the clinic. c. record the time that the stone passed. d. save a urine specimen to check for blood.

Answer: B Rationale: The patient should save the stone for analysis of the stone composition, which will help in determining treatment. Reporting the pain level and recording the time the stone passed are not essential. Hematuria is common with urinary calculi, so it is not necessary to test the urine for blood. Cognitive Level: Application Text Reference: p. 1173 Nursing Process: Implementation NCLEX: Physiological Integrity

12. A patient is admitted to the hospital with nephrotic syndrome after taking an OTC nonsteroidal antiinflammatory drug (NSAID) a week earlier. Which assessment data will the nurse expect to find related to this illness? a. Low blood pressure b. Recent weight gain c. Poor skin turgor d. High urine ketones

Answer: B Rationale: The patient with a rapid-onset nephrotic syndrome will have rapid weight gain associated with edema. Hypertension is a clinical manifestation of nephrotic syndrome. Skin turgor is normal because of the edema. Urine protein is high. Cognitive Level: Application Text Reference: p. 1167 Nursing Process: Assessment NCLEX: Physiological Integrity

33. Following a cystectomy, a patient has an ileal conduit created. The nurse identifies the nursing diagnosis of risk for infection related to altered urinary structures. An appropriate nursing intervention for this problem is to a. clamp the drainage bag while the patient sleeps. b. empty the drainage appliance every 2 to 3 hours or when it is one-third full. c. use liquid antiseptic in the appliance to decrease bacterial colonization. d. drain the conduit every 4 hours using a sterile catheter.

Answer: B Rationale: The patient with an ileal conduit will have an appliance to hold urine, which should be emptied to avoid reflux of urine back into the conduit. The drainage bag should not be clamped. The use of a liquid antiseptic will not decrease risk for infection. Unlike a continent pouch, the ileal conduit will drain continuously and is not drained with a catheter. Cognitive Level: Application Text Reference: p. 1193 Nursing Process: Implementation NCLEX: Physiological Integrity

34. Two days after surgery for an ileal conduit, the patient will not look at the stoma or participate in care. The patient insists that no one but the ostomy nurse specialist care for the stoma. The nurse identifies a nursing diagnosis of a. anxiety related to effects of procedure on lifestyle. b. disturbed body image related to change in body function. c. ineffective health maintenance related to refusal to participate in care. d. self-care deficit, toileting, related to denial of altered body function.

Answer: B Rationale: The patient's unwillingness to look at the stoma or participate in care indicates that disturbed body image is the best diagnosis. There are no data suggesting that the impact on lifestyle is a concern for the patient. The patient may be at risk for ineffective health maintenance if the lack of participation in care continues, but the patient's behavior is normal 2 days after surgery. The patient does not appear to be in denial. Cognitive Level: Application Text Reference: p. 1191 Nursing Process: Diagnosis NCLEX: Psychosocial Integrity

18. To prevent the recurrence of renal calculi, the nurse teaches the patient to a. avoid all sources of dietary calcium. b. drink diuretic fluids such as coffee. c. drink 2000 to 3000 ml of fluid a day. d. use a filter to strain all urine.

Answer: C Rationale: A fluid intake of 2000 to 3000 ml daily is recommended help flush out minerals before stones can form. Patients are not advised to avoid all calcium-containing foods and a high calcium intake may decrease the incidence of some types of stones. Coffee tends to increase stone recurrence. There is no need for a patient to strain all urine routinely after a stone has passed, and this will not prevent stones. Cognitive Level: Application Text Reference: p. 1172 Nursing Process: Implementation NCLEX: Physiological Integrity

8. When admitting a patient with acute glomerulonephritis, the nurse will ask the patient about a. history of high blood pressure. b. frequency of UTIs. c. recent sore throat and fever. d. family history of kidney disease.

Answer: C Rationale: Acute glomerulonephritis frequently occurs after a streptococcal infection such as strep throat. It is not caused by hypertension, UTI, or related to family history. Cognitive Level: Application Text Reference: p. 1165 Nursing Process: Assessment NCLEX: Physiological Integrity

10. A patient with nephrotic syndrome develops flank pain. The nurse will anticipate treatment with a. antibiotics. b. antihypertensives. c. anticoagulants. d. corticosteroids.

Answer: C Rationale: Flank pain in a patient with nephrosis suggests a renal vein thrombosis, and anticoagulation is needed. Antibiotics are used to treat a patient with flank pain caused by pyelonephritis. Antihypertensives are used if the patient has high blood pressure. Corticosteroids may be used to treat nephrotic syndrome but will not resolve a thrombosis. Cognitive Level: Application Text Reference: p. 1175 Nursing Process: Planning NCLEX: Physiological Integrity

38. Which information noted by the nurse when caring for a patient with a bladder infection is most important to report to the health care provider? a. Dysuria b. Temperature 100.1° F c. Left-sided flank pain d. Hematuria

Answer: C Rationale: Flank pain indicates that the patient may have developed pyelonephritis as a complication of the bladder infection. The other clinical manifestations are consistent with a lower UTI. Cognitive Level: Application Text Reference: p. 1161 Nursing Process: Assessment NCLEX: Physiological Integrity

19. In planning teaching for a patient with benign nephrosclerosis, the nurse should include instructions regarding a. measuring daily intake and output amounts. b. obtaining and documenting daily weights. c. monitoring and recording blood pressure. d. preventing bleeding caused by anticoagulants.

Answer: C Rationale: Hypertension is the major symptom of nephrosclerosis. Measurements of intake and output and daily weights are not necessary unless the patient develops renal insufficiency. Anticoagulants are not used to treat nephrosclerosis. Cognitive Level: Application Text Reference: p. 1175 Nursing Process: Planning NCLEX: Health Promotion and Maintenance

24. A 78-year-old patient is admitted to the hospital with dehydration and electrolyte imbalance. The patient is confused and incontinent of urine on admission. In developing a plan of care for the patient, an appropriate nursing intervention for the patient's incontinence is to a. insert an indwelling catheter. b. apply absorbent incontinent pads. c. assist the patient to the bathroom q2hr. d. restrict fluids after the evening meal.

Answer: C Rationale: In older or confused patients, incontinence may be avoided by using scheduled toileting times. Indwelling catheters increase the risk for UTI. Incontinent pads increase the risk for skin breakdown. Restricting fluids is not appropriate in a patient with dehydration. Cognitive Level: Application Text Reference: pp. 1183-1185 Nursing Process: Planning NCLEX: Physiological Integrity

4. To relieve the symptoms of a lower UTI for which the patient is taking prescribed antibiotics, the nurse suggests that the patient use the OTC urinary analgesic phenazopyridine (Pyridium) but cautions the patient that this preparation a. contains methylene blue, which turns the urine blue or green. b. should be taken on an empty stomach for maximum effect. c. causes the urine to turn reddish orange and can stain underclothing. d. frequently causes allergic reactions and should be stopped if a rash occurs.

Answer: C Rationale: Patients should be taught that Pyridium will color the urine deep orange and stain underclothing. Urised may turn the urine blue or green. The medication can cause gastrointestinal distress and should be taken with food. Although an allergic reaction may occur, this is not common. Cognitive Level: Comprehension Text Reference: p. 1158 Nursing Process: Implementation NCLEX: Physiological Integrity

9. The nurse establishes a nursing diagnosis of excess fluid volume related to inflammation at the glomerular basement membrane in a patient with acute glomerulonephritis. To best evaluate whether the problem identified in the nursing diagnosis has resolved, the nurse will monitor for a. proteinuria. b. elevated creatinine. c. periorbital edema. d. hematuria.

Answer: C Rationale: Resolution of the excess fluid volume is best evaluated by changes in edema. The other data may indicate whether the glomerulonephritis is resolving but do not provide data about fluid volume. Cognitive Level: Application Text Reference: p. 1165 Nursing Process: Evaluation NCLEX: Physiological Integrity

13. A 98-year-old patient with benign prostatic hyperplasia has a markedly distended bladder and is agitated and confused. All the following orders are received from the emergency department health care provider. Which order should the nurse act on first? a. Draw blood for blood urea nitrogen (BUN) and creatinine. b. Administer lorazepam (Ativan) 0.5 mg. c. Insert 16 French retention catheter. d. Schedule for IVP.

Answer: C Rationale: The patient's history and clinical manifestations are consistent with acute urinary retention, and the priority action is to relieve the retention by catheterization. The BUN and creatinine measurements can be obtained after the catheter is inserted. The patient's agitation may resolve once the bladder distention is corrected, and sedative drugs should be used cautiously in older patients. The IVP is an appropriate test, but does not need to be done urgently. Cognitive Level: Application Text Reference: p. 1185 Nursing Process: Implementation NCLEX: Physiological Integrity

31. A patient undergoing a left ureterolithotomy returns to the surgical unit with a left ureteral catheter and a urethral catheter in place. When caring for the patient, the nurse will plan to a. aspirate the ureteral catheter if output decreases. b. clamp the ureteral catheter unless output from the urethral catheter stops. c. keep the patient on bed rest until the ureteral catheter is discontinued. d. teach the patient about home care for both catheters.

Answer: C Rationale: To avoid displacing the ureteral catheter, the patient is usually on bed rest until the catheter is removed. Aspiration of the ureteral catheter might damage tissue in the renal pelvis. The catheter is not clamped. The patient is not usually discharged with a ureteral catheter in place. Cognitive Level: Application Text Reference: p. 1187 Nursing Process: Planning NCLEX: Physiological Integrity

20. A 32-year-old patient is diagnosed with polycystic kidney disease. Which information is most appropriate for the nurse to include in teaching at this time? a. Differences between hemodialysis and peritoneal dialysis b. Complications of renal transplantation c. Methods for treating chronic and severe pain d. Importance of genetic counseling

Answer: D Rationale: Because a 32-year-old patient may be considering having children, the nurse should include information about genetic counseling when teaching the patient. The well-managed patient will not need to choose between hemodialysis and peritoneal dialysis or know about the effects of transplantation for many years. There is no indication that the patient has chronic pain. Cognitive Level: Application Text Reference: p. 1176 Nursing Process: Implementation NCLEX: Health Promotion and Maintenance

6. A 72-year-old patient with benign prostatic hyperplasia and a history of frequent UTIs is admitted to the hospital with chills, fever, and nausea and vomiting. To determine whether the patient has an upper UTI, the nurse will assess for a. suprapubic pain. b. foul-smelling urine. c. bladder distension. d. costovertebral angle (CVA) tenderness.

Answer: D Rationale: CVA tenderness is characteristic of pyelonephritis. The other symptoms are characteristic of lower UTI and are likely to be present if the patient also has an upper UTI. Cognitive Level: Application Text Reference: p. 1161 Nursing Process: Assessment NCLEX: Physiological Integrity

22. When obtaining the health history for a 30-year-old patient who smokes two packs of cigarettes daily, the nurse will plan to do teaching about the increased risk for a. interstitial cystitis. b. UTI. c. kidney stones. d. bladder cancer.

Answer: D Rationale: Cigarette smoking is a risk factor for bladder cancer. The patient's risk for developing interstitial cystitis, UTI, or kidney stones will not be reduced by quitting smoking. Cognitive Level: Application Text Reference: p. 1178 Nursing Process: Planning NCLEX: Health Promotion and Maintenance

35. The nurse working in a urology clinic receives a call from a patient who had a transurethral resection with fulguration for bladder cancer 3 days previously. Which information given by the patient is of most concern to the nurse? a. The patient is voiding every 4 hours at night. b. The patient is using opioids for pain. c. The patient is very anxious about the cancer. d. There are clots in the urine.

Answer: D Rationale: Clots in the urine are not expected and require further follow-up. Voiding every 4 hours, use of opioids for pain, and anxiety are typical after this procedure. Cognitive Level: Application Text Reference: p. 1179 Nursing Process: Assessment NCLEX: Physiological Integrity

28. After the home health nurse teaches a patient with a neurogenic bladder how to use intermittent catheterization for bladder emptying, which patient statement indicates that the teaching has been effective? a. "I will need to buy seven new catheters weekly and use a new one every day." b. "I will use a sterile catheter and gloves for each time I self-catheterize." c. "I will need to take prophylactic antibiotics to prevent any urinary tract infections." d. "I will wash the catheter with soap and water before and after each catheterization."

Answer: D Rationale: Patients who are at home can use a clean technique for intermittent self-catheterization and change the catheter every 7 days. There is no need to use a new catheter every day, to use sterile catheters, or to take prophylactic antibiotics. Cognitive Level: Application Text Reference: p. 1188 Nursing Process: Evaluation NCLEX: Safe and Effective Care Environment

21. When assessing a patient who complains of a feeling of incomplete bladder emptying and a split, spraying urine stream, the nurse asks about a history of a. recurrent renal calculi. b. kidney trauma. c. bladder infection. d. gonococcal urethritis.

Answer: D Rationale: The patient's clinical manifestations are consistent with urethral strictures, a possible complication of gonococcal urethritis. These symptoms are not consistent with renal calculi, kidney trauma, or bladder infection. Cognitive Level: Application Text Reference: p. 1174 Nursing Process: Assessment NCLEX: Physiological Integrity

To evaluate the effectiveness of treatment for the patient with nephrotic syndrome, you assess the A. Blood pressure q4h B. Abdominal girth daily C. Urine of each voiding for protein D. Daily dietary protein intake

B. Abdominal girth daily A major nursing intervention for a patient with nephrotic syndrome is related to edema. It is important to assess the edema by weighing the patient daily, accurately recording intake and output, and measuring abdominal girth or extremity size. Comparing this information daily provides you with a tool for assessing the effectiveness of treatment.

patient's bowel sounds.

Before administering sodium polystyrene sulfonate (Kayexalate) to a patient with hyperkalemia, the nurse should assess the ______________

What are the immunologic mechanisms involved in glomerulonephritis? A. Tubular blocking by precipitates of bacteria and antibody reactions B. Deposition of immune complexes and complement along the glomerular basement membrane (GBM) C. Thickening of the GBM from autoimmune microangiopathic changes D. Destruction of glomeruli by proteolytic enzymes contained in the GBM

B. Deposition of immune complexes and complement along the glomerular basement membrane (GBM) All forms of immune complex disease are characterized by an accumulation of antigen, antibody, and complement in the glomeruli, which can result in tissue injury. The immune complexes activate complement. Complement activation results in the release of chemotactic factors that attract polymorphonuclear leukocytes, histamine, and other inflammatory mediators. The result of these processes is glomerular injury.

What is a priority nursing diagnosis for the patient with nephrotic syndrome? A. Imbalanced nutrition: less than body requirements B. Disturbed body image C. Decreased cardiac output D. Acute pain

B. Disturbed body image Support for the patient, in terms of coping with an altered body image, is essential because there is often embarrassment and shame associated with the edematous appearance.

Which medication usually is prescribed for patients with nephrotic syndrome? A. Sulfa B. Prednisone C. Amoxicillin D. Sulfisoxazole

B. Prednisone Corticosteroids and cyclophosphamide (Cytoxan) may be used for the treatment of severe cases of nephrotic syndrome. Prednisone has been effective to various degrees in persons with early-stage nephrosis, membranous glomerulonephritis, proliferative glomerulonephritis, and lupus nephritis.

What constituent is expected when evaluating the urinalysis of a patient with acute glomerulonephritis? A. Microscopic calculi B. Red blood cells and protein C. Escherichia coli D. Platelets

B. Red blood cells and protein Dipstick urinalysis and urine sediment microscopy reveal the presence of erythrocytes in significant numbers. Erythrocyte casts are highly suggestive of APSGN. Proteinuria may range from mild to severe. Screening blood tests include BUN and serum creatinine levels to assess the extent of renal impairment.

What is a priority nursing diagnosis for the patient with nephrotic syndrome? A. Activity intolerance B. Risk for infection C. Decreased cardiac output D. Imbalanced nutrition: less than body requirements

B. Risk for infection The patient with nephritic syndrome is susceptible to infection and should take measures to avoid exposure to persons known to have infections.

The mother of a child diagnosed with pyelonephritis asks if the kidneys were damaged because of this. What is the best response by the nurse? a) Yes, all children who get pyelonephritis have renal scarring. b) The child's risk for renal scarring is increased with pyelonephritis. c) As long as IV antibiotics are started, there is no risk of renal damage. d) No, if the child is urinating normally, the kidneys were not damaged.

B. The child's risk for renal scarring is increased with pyelonephritis. Correct Explanation: It would not be possible to determine if the child has renal scarring with pyelonephritis until more testing is performed. It can result in renal scarring with this type of problem, but that does not mean there will definitely be complications. Antibiotics are usually the treatment of choice in this situation, but it cannot be determined when the damage had occurred.

The diagnosis of urethral diverticula is confirmed by which diagnostic test? A. Blood urea nitrogen (BUN) B. Voiding cystourethrography (VCUG) C. Intravenous pyelogram (IVP) D. Cystoscopy

B. Voiding cystourethrography (VCUG) When palpated, urethral diverticula may be quite tender and express a purulent discharge through the urethra. Radiographic studies such as VCUG should be used to confirm the diagnosis. Additional studies include ultrasound and magnetic resonance imaging (MRI) to determine the size of the diverticulum in relation to the urethral lumen.

Which urinalysis result do you recognize as an abnormal finding? A. pH of 6.0 B. WBC count: 9/hpf C. Amber yellow color D. Specific gravity of 1.025

B. WBC count: 9/hpf Normal WBC levels in urine are below 5/hpf, and levels exceeding this indicate inflammation or urinary tract infection.

A patient with a ureterolithotomy returns from surgery with a nephrostomy tube in place. Postoperative nursing care of the patient includes A. encouraging the patient to drink fruit juices and milk B. forcing at least 2 to 3 L of fluids per day after nausea has subsided C. irrigating the nephrostomy tube with 10 mL of normal saline solution as needed D. notifying the physician if nephrostomy tube drainage is more than 30 mL per hour

B. forcing at least 2 to 3 L of fluids per day after nausea has subsided The nephrostomy tube is inserted directly into the pelvis of the kidney and attached to connecting tubing for closed drainage. The catheter should never be kinked, compressed, or clamped. If the patient complains of excessive pain in the area or if there is excessive drainage around the tube, check the catheter for patency. If irrigation is ordered, strict aseptic technique is required. No more than 5 mL of sterile saline solution is gently instilled at one time to prevent overdistention of the kidney pelvis and renal damage. Infection and secondary stone formation are complications associated with the insertion of a nephrostomy tube. Patients should drink 2 to 3 liters of fluid per day to reduce risk of infection and stone formation.

You identify a risk factor for kidney and bladder cancer in a patient who relates a history of A. aspirin use. B. tobacco use. C. chronic alcohol abuse. D. use of artificial sweeteners.

B. tobacco use. Cigarette smoking is the most significant risk factor for renal cell carcinoma. An increased incidence also has been seen among first-degree relatives. Other risk factors include obesity, hypertension, and exposure to asbestos, cadmium, and gasoline. The risk of renal cancer is increased for individuals who have acquired cystic disease of the kidney associated with end-stage renal disease. Risk factors for bladder cancer include smoking, exposure to dyes used in the rubber and cable industries, and chronic abuse of phenacetin-containing analgesics.

What is a common urinary diversion that results in a stoma and the need for an ostomy pouch? A. Indian pouch B. Kock reservoir C. Ileal or colon conduit D. Continent ileal reservoir

C. Ileal or colon conduit The most commonly performed incontinent urinary diversion procedure is the ileal conduit (ileal loop). In this procedure, a 6- to 8-inch (15- to 20-cm) segment of the ileum is converted into a conduit for urinary drainage. The colon (colon conduit) can be used instead of the ileum. The ureters are anastomosed into one end of the conduit, and the other end of the bowel is brought out through the abdominal wall to form a stoma.

Why are renal tumors often difficult to diagnose? A. They do not show up on routine radiologic examinations. B. They mimic the pain associated with renal stones. C. They lack significant clinical manifestations. D. They are rare and occur unilaterally.

C. They lack significant clinical manifestations. Because there are no characteristic early symptoms of kidney cancer, many patients go undiagnosed until the disease has significantly progressed.

The most critical intervention in the prevention of renal calculi is for the patient to A. urinate frequently. B. eat a diet high in protein. C. maintain a high fluid intake. D. eliminate all calcium from the diet.

C. maintain a high fluid intake. The most important factor in the prevention of renal calculi is keeping urine dilute and free flowing. This reduces the risk of recurrent stone formation in many individuals. This is accomplished by drinking about 2000 to 2200 mL/day, with the residual 20% to 30% of fluids gained through consumption of foods. The volume of fluids is higher in the highly active patient who works outdoors or who regularly engages in demanding athletic activities.

The nursing activity "encourage increased fluid intake" is beneficial to patients with any type of urinary tract infection because it A. decreases residual urine. B. prevents urinary reflux. C. prevents urinary stasis. D. alters urinary pH.

C. prevents urinary stasis. Acute intervention for a patient with a UTI includes ensuring adequate fluid intake if it is not contraindicated. It is sometimes difficult to get the patient to maintain an adequate fluid intake because the person may think it will worsen the discomfort and frequency associated with a UTI. Tell patients that fluids will increase frequency of urination at first but will also dilute the urine, making the bladder less irritable. Fluids help to flush out bacteria before they have a chance to colonize in the bladder.

The nurse is caring for a child with nephrotic syndrome. The child is noted to have edema. The edema would most likely be seen where on this child? a) Sacrum b) Abdomen c) Eyes d) Fingers

Eyes Correct Explanation: Edema is usually the presenting symptom in nephrotic syndrome, appearing first around the eyes and ankles.

dialysate is infused into the abdomen and remains there 2-6 hours. The dialysate is removed by gravity drainage after the prescribed time

Continuous Ambulatory Peritoneal Dialysis (CAPD)

A symptom often seen in acute glomerulonephritis is edema. The most common site the edema is first noted is in which area of the body? a) Sacrum b) Eyes c) Hands d) Ankles

Eyes Correct Explanation: Periorbital edema may accompany or precede hematuria in children with acute glomerulonephritis. Edema in the ankles, hands and sacrum are not noted in acute glomerulonephritis.

A school nurse is trying to prevent poststreptococcal glomerulonephritis in children. Which of the following would be the best way to prevent this? a) Prophylactic antibiotics after strep throat are important. b) Tell parents to give ibuprofen if their child has a sore throat. c) All children in the child's class should be tested for strep throat if there is a positive. d) Encourage the child to take all the antibiotics if diagnosed with strep throat.

D)Encourage the child to take all the antibiotics if diagnosed with strep throat. Correct Explanation: Encouraging the child to take all the antibiotics if diagnosed with strep throat is important. It is not necessary to test the people in the community that the child came in contact with unless they are symptomatic. Ibuprofen does not cure strep throat and that is what usually causes poststreptococcal glomerulonephritis. Prophylactic antibiotics after a strep infection are not necessary.

You are providing care for a patient who has been admitted to the hospital for the treatment of nephrotic syndrome. What is a priority nursing assessment in the care of this patient? A. Assessment of pain and level of consciousness B. Assessment of serum calcium and phosphorus levels C. Blood pressure and assessment for orthostatic hypotension D. Daily weights and measurement of the patient's abdominal girth

D. Daily weights and measurement of the patient's abdominal girth Peripheral edema is characteristic of nephrotic syndrome, and a key nursing responsibility in the care of patients with the disease is close monitoring of abdominal girth, weight, and extremity size. Pain, level of consciousness, and blood pressure are less important in the care these patients. Abnormal calcium and phosphorus levels are not commonly associated with nephrotic syndrome.

What causes the edema that occurs in nephrotic syndrome? A. Increased hydrostatic pressure caused by sodium retention B. Decreased aldosterone secretion from adrenal insufficiency C. Increased fluid retention caused by decreased glomerular filtration D. Decreased colloidal osmotic pressure caused by loss of serum albumin

D. Decreased colloidal osmotic pressure caused by loss of serum albumin The increased glomerular membrane permeability found in nephrotic syndrome is responsible for the massive excretion of protein in the urine. This results in a decreased serum protein level and subsequent edema formation. Ascites and anasarca (massive generalized edema) develop if there is severe hypoalbuminemia.

The nurse providing education for the patient with polycystic kidney disease (PKD) should include what in the teaching plan? A. Measurement of blood pressure B. Performance of sterile self-catheterization C. Avoidance of beverages containing caffeine D. Early recognition of the signs of infection or bleeding

D. Early recognition of the signs of infection or bleeding There is no specific treatment for PKD. A major aim of treatment is to prevent infections of the urinary tract and to treat them with appropriate antibiotics if they occur.

Which factor in the patient's history is most commonly associated with nephrosclerosis? A. Renal calculi B. Frequent UTIs C. Congenital nephritis D. Essential hypertension

D. Essential hypertension Nephrosclerosis is caused by vascular changes resulting from hypertension and from the atherosclerosis process.

Which assessment finding is associated with renal calculi? A. Urinary bladder distention B. Hypertension C. Proteinuria D. Flank pain

D. Flank pain Clinical manifestations of renal calculi include abdominal or flank pain (typically severe), hematuria, and renal colic.

Which clinical findings are consistent with the diagnosis of acute pyelonephritis? A. Hypertension and costovertebral angle tenderness B. Increased blood urea nitrogen and fatigue C. Decreased serum creatinine and chills D. Leukocytosis and flank pain

D. Leukocytosis and flank pain The clinical manifestations of acute pyelonephritis vary from mild fatigue to the sudden onset of chills, fever, vomiting, malaise, flank pain, and the lower urinary tract symptoms characteristic of cystitis, including dysuria, urinary urgency, and frequency. Costovertebral tenderness (costovertebral angle [CVA] pain) typically occurs on the affected side. A complete blood cell count shows leukocytosis and a shift to the left, with an increase in the number of immature neutrophils (bands).

In teaching a patient with pyelonephritis about the disorder, you inform him that the organisms that cause pyelonephritis most commonly reach the kidneys through A. the bloodstream. B. the lymphatic system. C. an descending infection. D. an ascending infection.

D. an ascending infection. The organisms that usually cause urinary tract infections (UTIs) are introduced by the ascending route from the urethra and originate in the perineum.

You identify a risk for urinary calculi in a patient who relates a health history that includes A. adrenal insufficiency. B. serotonin deficiency. C. hyperaldosteronism. D. hyperparathyroidism.

D. hyperparathyroidism. Excessive levels of circulating parathyroid hormone (PTH) usually lead to hypercalcemia and hypophosphatemia. In the kidneys excess calcium in the filtrate cannot be reabsorbed, leading to increased levels of calcium in the urine (hypercalciuria). This increased urinary calcium level along with a large amount of urinary phosphate can lead to calculi formation.

When developing an expected outcome for the patient with urinary retention, you would include A. freedom from bladder fullness. B. patient who verbalizes need to void. C. fluid intake less than 1200 mL/day. D. non-palpable bladder after voiding.

D. non-palpable bladder after voiding. If the bladder is emptied fully, you should not be able to palpate it.

You obtain a history from the mother of a child with glomerulonephritis about how he became ill. Which of the following would you expect her to tell you she noticed? a) Headache, loss of appetite b) Dark brown urine c) Loss of weight, oliguria d) Diuresis and pallor

Dark brown urine Correct Explanation: Acute glomerulonephritis often presents with glomeruli bleeding, which is revealed as black or brown urine from old blood.

- diuresis up to 10L/day - urine is very dilute - BUN stops increasing - monitored for hypokalemia and hyponatremia

Diuretic phase

ibuprofen (Motrin)

During assessment of a patient with decreased renal function, which of these medications taken by the patient at home will be of most concern to the nurse?

Check the patient's blood pressure.

During hemodialysis, a patient complains of nausea and dizziness. Which action should the nurse take first?

A nurse is developing a teaching plan for the parents of an 8 year old experiencing nocturnal enuresis. The nurse determines that additional teaching is needed when the parents identify which of the following as an appropriate measure? a) Engaging the child in stress reduction measures b) Giving desmopressin intranasally c) Encouraging fluid intake after dinner d) Practicing bladder-stretching exercises

Encouraging fluid intake after dinner Correct Explanation: In many children, limiting fluids after dinner can be helpful for nocturnal enuresis. Bladder-stretching exercises also can be helpful. If these measures are ineffective, desmopressin may be prescribed. Stress factors may be contributing to the child's problem. Therefore, measures to address stress and promote coping would be appropriate.

inflammatory reaction in the glomerulus most commonly as a result of an antigen-antibody response to beta hemolytic streptocci

Glomerulonephritis

A nurse is performing an assessment on a child. Which of the following would be indicative of a potential for a urinary tract infection? a) Holding urine while at school. b) Not using cleansing towelettes routinely. c) Washing the genital area with water daily. d) Not using soap when cleaning the urethral area.

Holding urine while at school. Correct Explanation: UTIs are often caused by children who do not urinate frequently at school. It is important for a child to avoid using towelettes and soap in the genital area because this can increase the chance of a UTI. Washing the genital area with water daily does not increase the chance of a UTI.

Position one hand flat at the costovertebral angle (CVA) and strike it with the other fist.

How will the nurse assess the flank area of a patient with pyelonephritis for tenderness?

The nurse is assessing a male neonate and notes that the urethral opening is on the ventral aspect of the penis. The nurse documents this finding as which of the following? a) Bladder exstrophy b) Patent urachus c) Epispadias d) Hypospadias

Hypospadias Correct Explanation: Hypospadias is a condition in which the urethral opening in on the ventral surface of the penis. Epispadias is present when the urethral opening is on the dorsal surface of the penis. Patent urachus refers to a fistula between the bladder and umbilicus. Bladder exstrophy involves the bladder lying open and exposed on the abdomen.

Place the patient on bed rest.

In a patient with acute kidney injury (AKI) who requires hemodialysis, a temporary vascular access is obtained by placing a catheter in the left femoral vein. Which intervention will be included in the plan of care?

teach the patient to clean the urethral area, void a small amount into the toilet, and then void into a sterile specimen cup.

The health care provider orders a clean-catch urine specimen for culture and sensitivity testing for a patient with a suspected urinary tract infection (UTI). To obtain the specimen, the nurse will plan to

Most urinary tract infections seen in children are caused by which of the following? a) Hereditary causes b) Intestinal bacteria c) Dietary insufficiencies d) Fungal infections

Intestinal bacteria Correct Explanation: Although many different bacteria may infect the urinary tract, intestinal bacteria, particularly Escherichia coli, account for about 80% of acute episodes. Hereditary and dietary concerns are not causes of urinary tract infections.

The nurse is caring for an 8-year-old hospitalized child with nephrotic syndrome. Which of the following nursing interventions would be appropriate for this child? a) Test the urine for ketones twice a day b) Weigh the child once a week. c) Administer antipyretics as needed. d) Measure the abdominal girth daily.

Measure the abdominal girth daily. Correct Explanation: Measure the child's abdomen daily at the level of the um bilicus, and make certain that all staff personnel measure at the same level. Weigh the child at the same time every day on the same scale in the same clothing. Test the urine regularly for albumin and specific gravity. Elevated temperature is not an issue with nephrotic syndrome.

- urinary output decreases to less than 400 ml per 24 hours - increase in BUN, creatinine, uric acid, potassium, and magnesium levels and presence of metabolic acidosis - 1-3 week duration

OLIGURIC PHASE

utilization of the peritoneal cavity and peritoneum as the semipermeable membrane that removes excess fluid

Peritoneal dialysis

The nurse is collecting data on a 6-year-old child admitted with a possible urinary tract infection. Which of the following vital signs might indicate the possibility of an infection? a) Respirations 22 per minute b) Blood Pressure 100/70 c) Pulse rate 135 bpm d) Pulse oximetry 93% on room air

Pulse rate 135 bpm Correct Explanation: Data to collect regarding the child includes temperature, pulse (be alert for tachycardia) and respiration rates; normal vital signs for a 6-year-old would be a pulse rate of 70 to 115 beats per minute, so this rate shows tachycardia,. The other vital signs are all within normal limits for this age child.

The nurse is teaching a group of nursing students about genitourinary conditions. The nurse tells these students about a condition that occurs when there is an inflammation of the kidney and renal pelvis. The condition the nurse is referring to is which of the following? a) Ascites b) Oliguria c) Pyelonephritis d) Amenorrhea

Pyelonephritis Correct Explanation: Pyelonephritis is an inflammation of the kidney and renal pelvis. Oliguria is a subnormal volume of urine. Amenorrhea is the absence of menstruation. Ascites is edema in the peritoneal cavity.

A child diagnosed acute glomerulonephritis will most likely have a history of which of the following? a) Recent illness such as strep throat b) Hemorrhage or history of bruising easily c) Sibling diagnosed with the same disease d) Hearing loss with impaired speech development

Recent illness such as strep throat Correct Explanation: Symptoms of acute glomerulonephritis often appear one to three weeks after the onset of a streptococcal infection such as strep throat.

GFR increases during this phase of ARF

Recovery

The nurse is caring for a child admitted with acute glomerulonephritis. Which of the following clinical manifestations would likely have been noted in the child with this diagnosis? a) Smoky colored urine b) Jaundiced skin c) Strawberry red tongue d) Loose, dark stools

Smoky colored urine Explanation: The presenting symptom in acute glomerulonephritis is grossly bloody urine. The caregiver may describe the urine as smoky or bloody. Periorbital edema may accompany or precede hematuria. Loose stools are seen in diarrhea. A strawberry colored tongue is a symptom seen in the child with Kawasaki disease. Jaundiced skin is noted in Hepatitis.

The caregiver of a child being treated at home for acute glomerulonephritis calls the nurse reporting that her daughter has just had a convulsion. The child is resting comfortably but the caregiver would like to know what to do. The nurse would instruct the caregiver to do which of the following actions? a) Give the child fluids and report back to the nurse in a few hours. b) Take the child's blood pressure and report the findings to the nurse while the nurse is still on the phone. c) Give the child a diuretic and report back to the nurse in a few hours. d) Weigh the child in the same clothes she had been weighed in the day before and report the two weighs to the nurse while the nurse is on the phone.

Take the child's blood pressure and report the findings to the nurse while the nurse is still on the phone. Correct Explanation: Blood pressure should be monitored regularly using the same arm and a properly fitting cuff. If hypertension develops, a diuretic may help reduce the blood pressure to normal levels. An antihypertensive drug may be added if the diastolic pressure is 90 mm Hg or higher. The concern is immediate so reporting the findings in a few hours could delay needed treatment. The child should be weighed daily in the same clothes and using the same scale, but the blood pressure is the priority in this situation.

To prevent further urinary tract infections in a preschooler, what measures would you teach her mother? a) Teach her to take frequent tub baths to clean her perineal area. b) Teach her to wipe her perineum front to back after voiding. c) Suggest she drink less fluid daily to concentrate urine. d) Encourage her to be more ambulatory to increase urine output

Teach her to wipe her perineum front to back after voiding. Correct Explanation: Escherichia coli can be easily spread from the rectum to the urinary meatus and cause infection if girls do not take precautions against this.

The LPN/LVN gives the iron supplement and phosphate binder with lunch.

The RN observes an LPN/LVN carrying out all of the following actions while caring for a patient with stage 2 chronic kidney disease. Which action requires the RN to intervene?

The nurse knows which of the following is a description of peritoneal dialysis when compared to hemodialysis? a) The child must go into a facility to get peritoneal dialysis. b) There are strict diet and fluid restrictions. c) The child can live a more normal lifestyle. d) Therapy is only 3 to 4 days per week.

The child can live a more normal lifestyle. Correct Explanation: The child can live a more normal lifestyle with peritoneal dialysis. This is a 7-day-a-week procedure, but there are less diet restrictions and more freedom with this type of procedure. Peritoneal dialysis can be performed at home.

Serum potassium level 6.5 mEq/L

When the nurse is caring for a patient who has been admitted with a severe crushing injury after an industrial accident, which laboratory result will be most important to report to the health care provider?

60, The creatinine clearance approximates the GFR.

The result of a patient's creatinine clearance test is 60 mL/min. The nurse equates this finding to a glomerular filtration rate (GFR) of _____ mL/min.

have 2000 to 3000 mL of fluid a day.

To prevent the recurrence of renal calculi, the nurse teaches the patient to _____________

disturbed body image related to change in body function.

Two days after surgery for an ileal conduit, the patient will not look at the stoma or participate in care. The patient insists that no one but the ostomy nurse specialist care for the stoma. The nurse identifies a nursing diagnosis of __________

In caring for a child with nephrotic syndrome, which of the following interventions will be included in the child's plan of care? a) Ambulating three to four times a day b) Testing the urine for glucose levels regularly c) Increasing fluid intake by 50 cc an hour d) Weighing on the same scale each day

Weighing on the same scale each day Correct Explanation: The child with nephrotic syndrome is weighed every day using the same scale to accurately monitor the child's fluid gain and loss.

More protein will be allowed because of the removal of urea and creatinine by dialysis.

When a patient who has had progressive chronic kidney disease (CKD) for several years is started on hemodialysis, which information about diet will the nurse include in patient teaching?

recent sore throat and fever.

When admitting a patient with acute glomerulonephritis, it is most important that the nurse ask the patient about ________________

gonococcal urethritis.

When assessing a 30-year-old man who complains of a feeling of incomplete bladder emptying and a split, spraying urine stream, the nurse asks about a history of ______________

pain with urination.

When assessing the patient who has a lower urinary tract infection (UTI), the nurse will initially ask about _____________

Place the patient on a cardiac monitor.

When caring for a dehydrated patient with acute kidney injury who is oliguric, anemic, and hyperkalemic, which of the following prescribed actions should the nurse take first?

Check the fistula site for a bruit and thrill.

When caring for a patient with a left arm arteriovenous fistula, which action will the nurse include in the plan of care to maintain the patency of the fistula?

monitoring and recording blood pressure.

When planning teaching for a patient with benign nephrosclerosis the nurse should include instructions regarding ________________

"Do you have any pain when you urinate?"

When reading a patient's chart, the nurse notes that the patient has dysuria. To assess whether there is any improvement, which question will the nurse ask?

WBC: 20-26/hpf

When reviewing the results of a patient's urinalysis, which information indicates that the nurse should notify the health care provider?

"Your doctor will insert a lighted tube into the bladder through your urethra, inspect the bladder, and instill a dye that will outline your bladder on x-ray."

When teaching a patient scheduled for a cystogram via a cystoscope about the procedure, the nurse tells the patient, ________________

how often to evaluate venipuncture insertion site

every 15-30 min after the procedure to assess for bleeding

circulation of pt's blood with dialysate fluid, removing excess body fluid

hemodialysis

* what might occur if dialysate is left in the peritoneal cavity too long?

hyperglycemia


संबंधित स्टडी सेट्स

Chapter 6: Capacitors and Inductors

View Set

Wyoming Statues, Rules and Regulations Common to All Lines

View Set

3.10.2 Erosion - Mass Movements & Gravity

View Set

Chapter 50: Nursing Care of the Child With an Alteration in Behavior, Cognition, Development, or Mental Health/Cognitive or Mental Health Disorder

View Set